Physeo Microbiology [1 ed.]

MEDICAL COURSE AND STEP 1 REVIEW FIRST EDITION Accompanies online videos taught by Rhett Thomson & Michael Christen

2,750 502 102MB

English Pages 358 Year 2018

Report DMCA / Copyright

DOWNLOAD FILE

Polecaj historie

Physeo Microbiology [1 ed.]

  • Commentary
  • Sold at https://physeo.com/shop/

Table of contents :
MICROBIOLOGY FUNDAMENTALS
Section 1 - Bacterial Structures
Section 2 - Virulence
Section 3 - Bacterial Genetics
Section 4 - Prion Diseases
Section 5 - Introduction to Fungi
Section 6 - Viral Structures
Section 7 - Viral Genetics
Section 8 - Viral Genomes and Replication
BACTERIA
Section 1 - Clostridium tetani
Section 2 - Clostridium botulinum
Section 3 - Clostridium difficile
Section 4 - Clostridium perfringens
Section 5 - Bacillus anthracis
Section 6 - Bacillus cereus
Section 7 - Listeria monocytogenes
Section 8 - Corynebacterium diphtheriae
Section 9 - Viridans streptococci
Section 10 - Streptococcus pneumoniae
Section 11 - Streptococcus pyogenes
Section 12 - Streptococcus agalactiae
Section 13 - Streptococcus gallolyticus
Section 14 - Enterococci
Section 15 - Staphylococcus aureus
Section 16 - Staphylococcus epidermidis
Section 17 - Staphylococcus saprophyticus
Section 18 - Nocardia
Section 19 - Actinomyces israelii
Section 20 - Neisseria overview
Section 21 - Neisseria meningitidis
Section 22 - Neisseria gonorrhoeae
Section 23 - Moraxella catarrhalis
Section 24 - Haemophilus influenzae
Section 25 - Bordetella pertussis
Section 26 - Pasteurella multocida, Brucella spp., and Francisella tularensis
Section 27 - Salmonella spp.
Section 28 - Salmonella typhi
Section 29 - Shigella
Section 30 - E. coli
Section 31 - Klebsiella
Section 32 - Proteus mirabilis
Section 33 - Pseudomonas aeruginosa
Section 34 - Yersinia pestis and enterocolitica
Section 35 - Enterobacter, Citrobacter, & Serratia marcescens
Section 36 - Bartonella henselae
Section 37 - Legionella pneumophila
Section 38 - Campylobacter jejuni
Section 39 - Vibrio cholerae & Vibrio vulnificus
Section 40 - Helicobacter pylori
Section 41 - Mycobacterium tuberculosis
Section 42 - Mycobacterium leprae
Section 43 - Mycobacterium avium-intracellulare, Scrofulaceum, and Marinum
Section 44 - Borrelia burgdorferi
Section 45 - Leptospira interrogans
Section 46 - Treponema pallidum
Section 47 - Chlamydia trachomatis, Chlamydophila pneumoniae, and Chlamydophila psittaci
Section 48 - Coxiella burnetii
Section 49 - Gardnerella vaginalis
Section 50 - Mycoplasma pneumoniae
Section 51 - Rickettsia prowazekii, Rickettsia rickettsii, and Rickettsia typhi
Section 52 - Ehrlichia & Anaplasma
Section 53 - Miscellaneous Organisms
FUNGI
Section 1 - Histoplasma capsulatum
Section 2 - Blastomyces dermatitidis
Section 3 - Coccidioides immitis
Section 4 - Paracoccidioides brasiliensis
Section 5 - Microsporum, Trichophyton, and Epidermophyton
Section 6 - Malassezia furfur
Section 7 - Sporothrix schenckii
Section 8 - Candida albicans
Section 9 - Aspergillus fumigatus
Section 10 - Cryptococcus neoformans
Section 11 - Mucor & Rhizopus spp.
Section 12 - Pneumocystis jirovecii
PARASITES
Section 1 - Giardia lamblia
Section 2 - Entamoeba histolytica
Section 3 - Cryptosporidium
Section 4 - Toxoplasma gondii
Section 5 - Naegleria fowleri
Section 6 - Trypanosoma brucei, gambiense, and rhodesiense
Section 7 - Malaria
Section 8 - Babesia
Section 9 - Trypanosoma cruzi
Section 10 - Leishmania donovani
Section 11 - Trichomonas vaginalis
Section 12 - Enterobius vermicularis (pinworm)
Section 13 - Ascaris lumbricoides (giant roundworm)
Section 14 - Strongyloides stercoralis (threadworm)
Section 15 - Ancylostoma duodenale and Necator americanus (hookworms)
Section 16 - Trichinella spiralis
Section 17 - Trichuris trichiura (whipworm)
Section 18 - Toxocara canis
Section 19 - Onchocerca volvulus
Section 20 - Loa loa and Wuchereria bancrofti
Section 21 - Taenia solium
Section 22 - Diphyllobothrium latum
Section 23 - Echinococcus granulosus
Section 24 - Schistosoma haematobium, mansoni, and japanicom
Section 25 - Clonorchis sinensis
Section 26 - Sarcoptes scabiei
Section 27 - Pediculus corporis, pubis and capitis
VIRUSES
Section 1 - Herpes simplex virus-1 (HSV-1) and herpes simplex virus-2 (HSV-2)
Section 2 - Varicella-zoster virus (VZV)
Section 3 - Epstein-Barr Virus (EBV)
Section 4 - Cytomegalovirus (CMV)
Section 5 - Roseola (HHV-6 and HHV-7)
Section 6 - Kaposi sarcoma (HHV-8)
Section 7 - Hepatitis B Virus (HBV)
Section 8 - Smallpox, Cowpox, and Molluscum contagiosum
Section 9 - Human papillomavirus
Section 10 - Adenovirus
Section 11 - JC Virus and BK Virus
Section 12 - Parvovirus B19
Section 13 - Coronavirus and SARS
Section 14 - Human immunodeficiency virus (HIV)
Section 15 - Human T-lymphotropic virus (HTLV)
Section 16 - Hepatitis C Virus (HCV)
Section 17 - Yellow fever
Section 18 - Dengue Fever
Section 19 - West Nile Virus and St. Louis Encephalitis
Section 20 - Zika Virus
Section 21 - Rubella
Section 22 - Eastern, Western, and Venezuelan equine encephalitis viruses; Chikungunya virus
Section 23 - California Encephalitis
Section 24 - Rift Valley Fever and Crimean-Congo Hemorrhagic Fever Virus
Section 25 - Hantavirus
Section 26 - Influenza virus
Section 27 - Lassa fever encephalitis and Lymphocytic choriomeningitis virus
Section 28 - Hepatitis D Virus
Section 29 - Parainfluenza Virus
Section 30 - RSV and hMPV
Section 31 - Measles
Section 32 - Mumps
Section 33 - Rabies Virus
Section 34 - Ebola Virus and Marburg Virus
Section 35 - Norovirus
Section 36 - Hepatitis E virus
Section 37 - Rotavirus and Colorado Tick Fever
Section 38 - Poliovirus
Section 39 - Echovirus
Section 40 - Rhinovirus
Section 41 - Coxsackieviruses (A & B)
Section 42 - Hepatitis A Virus
Table 5.1.1 - Bacterial structures
Figure 5.1.1 - Gram positive bacterial envelope
Figure 5.1.2 - Gram negative bacterial envelope
Figure 5.1.3 - Envelope of Mycoplasma spp.
Figure 5.1.4 - Envelope of Mycobacteria spp.
Figure 5.1.5 - Acid-fast stain of Mycobacterium Tuberculosis
Figure 5.1.6 - Bacterial endospore
Table 5.1.2 - Overview of virulence factors
Figure 5.1.7 - Protein A mechanism
Figure 5.1.8 - Visual demonstration of Type III secretion systems
Table 5.1.3 - Bacterial exotoxins
Figure 5.1.9 - Exotoxins that inhibit protein synthesis
Figure 5.1.10 - Exotoxins that increase fluid secretion using adenylate cyclase
Figure 5.1.11 - Exotoxin (heat-stable toxin) that increases fluid secretion through guanylate cyclase
Figure 5.1.12 - Pertussis toxin mechanism
Figure 5.1.13 - Mechanism of tetanospasmin
Figure 5.1.14 - Mechanism of botulinum toxin
Figure 5.1.15 - Mechanism of exotoxins that disrupt cell membranes
Figure 5.1.16 - Mechanism of superantigens
Table 5.1.4 - Mechanisms of genetic change in bacteria
Figure 5.1.17 - Random mutation in bacteria
Figure 5.1.18 - Phase variation in bacteria
Figure 5.1.19 - Transformation in bacteria
Figure 5.1.20 - Conjugation in bacteria
Figure 5.1.21 - Generalized transduction (lytic cycle)
Figure 5.1.22 - Specialized transduction (lysogenic cycle)
Figure 5.1.23 - Restriction modification
Figure 5.1.24 - Brain biopsy with vacuolations from spongiform encephalopathy
Figure 5.1.25 - Fungi structure
Figure 5.1.26 - Filamentous fungus
Figure 5.1.27 - Mold (left) and yeast (right)
Figure 5.1.28 - Spores of Aspergillus
Figure 5.1.29 - Enveloped virus structures
Figure 5.1.30 - Naked virus structures
Figure 5.1.31 - Adsorption, penetration, and uncoating of naked viruses
Figure 5.1.32 - Adsorption, penetration, and uncoating of enveloped viruses
Figure 5.1.33 - Bacteriophage structure
Table 5.1.5 - Genetic adjustments and activities in viruses
Figure 5.1.34 - Genetic shift and genetic drift
Figure 5.1.35 - Phenotypic mixing
Figure 5.1.36 - HDV superinfection
Figure 5.1.37 - Positive-sense and negative-sense viruses
Figure 5.1.38 - Poxvirus cytoplasm replication
Figure 5.1.39 - HBV structure
Figure 5.1.40 - RNA virus replication
Figure 5.1.41 - DNA virus replication
Figure 5.2.1 - Bacteria overview figure
Figure 5.2.2 - Gram stain
Figure 5.2.3 - Gram-positive bacilli
Figure 5.2.4 - Tetanospasmin
Figure 5.2.5 - Gram stain of Clostridium botulinum
Figure 5.2.6 - Gram stain of Clostridium difficile
Figure 5.2.7 - Pseudomembranous colitis
Figure 5.2.8 - Gram stain of Clostridium perfringens
Figure 5.2.9 - Gas gangrene
Figure 5.2.10 - Gram stain of Bacillus anthracis
Figure 5.2.11 - Cutaneous anthrax
Figure 5.2.12 - Gram stain of Bacillus cereus
Figure 5.2.13 - Gram stain of gram-positive rods
Figure 5.2.14 - Hemolysis on 5% sheep blood agar
Figure 5.2.15 - Catalase test
Figure 5.2.16 - Gram stain of Corynebacterium diphtheriae
Figure 5.2.17 - Cystine-tellurite agar
Figure 5.2.18 - Pseudomembranous pharyngitis
Figure 5.2.19 - Gram stain of Streptococci
Figure 5.2.20 - Microbial resistance and sensitivity to various compounds
Figure 5.2.21 - Splinter hemorrhages
Figure 5.2.22 - Quellung reaction
Figure 5.2.23 - Otitis media
Figure 5.2.24 - Cellulitis
Figure 5.2.25 - Impetigo
Figure 5.2.26 - Erysipelas
Figure 5.2.27 - Sandpaper-like rash
Figure 5.2.28 - Strawberry tongue
Figure 5.2.29 - GAS pharyngitis
Figure 5.2.30 - CAMP test
Figure 5.2.31 - Gram stain of Enterococci
Figure 5.2.32 - Gram stain of Staphylococcus
Figure 5.2.33 - Mannitol fermentation test
Figure 5.2.34 - Coagulase test tube
Figure 5.2.35 - Urease test
Figure 5.2.36 - Gram stain of Nocardia
Figure 5.2.37 - Acid-fast stain of Mycobacterium Tuberculosis (similar to Nocardia)
Figure 5.2.38 - Gram stain of Actinomyces
Figure 5.2.39 - Sulfur granule
Figure 5.2.40 - Gram stain of Neisseria
Figure 5.2.41 - Chocolate agar
Figure 5.2.42 - Gram negative bacterial cell wall
Figure 5.2.43 - Oxidase test
Figure 5.2.44 - Chocolate agar and Thayer-Martin agar
Figure 5.2.45 - Gram stain of Moraxella
Figure 5.2.46 - Otoscopic view of otitis media
Figure 5.2.47 - Gram Stain of Haemophilus influenzae
Figure 5.2.48 - Satellite colonies of Haemophilus influenzae growing near Staphylococcus aureus on blood agar
Figure 5.2.49 - Epiglottis
Figure 5.2.50 - Thumb sign
Figure 5.2.51 - Cherry red epiglottis
Figure 5.2.52 - Gram stain of Bordetella
Figure 5.2.53 - Regan-Lowe agar
Figure 5.2.54 - Gram stain of gram-negative coccobacilli
Figure 5.2.55 - Granuloma
Figure 5.2.56 - Gram stain of Salmonella
Figure 5.2.57 - TSI agar
Figure 5.2.58 - Peyer’s patches
Figure 5.2.59 - Rose spots
Figure 5.2.60 - Gram stain of Shigella
Figure 5.2.61 - Actin
Figure 5.2.62 - Neutrophils
Figure 5.2.63 - Schistocytes
Figure 5.2.64 - Gram stain of E. coli
Figure 5.2.65 - Eosin methylene blue aga
Figure 5.2.66 - MacConkey agar
Figure 5.2.67 - Indole test
Figure 5.2.68 - Lipopolysaccharide
Figure 5.2.69 - Mucoid colonies
Figure 5.2.70 - Struvite crystals
Figure 5.2.71 - Staghorn calculus
Figure 5.2.72 - Gram stain of Pseudomonas
Figure 5.2.73 - Pyoverdin and pyocyanin (bluish-green color) of Pseudomonas (left)
Figure 5.2.74 - Corneal abrasion
Figure 5.2.75 - Otitis externa
Figure 5.2.76 - Hot tub folliculitis
Figure 5.2.77 - Gram stain gram of Yersinia enterocolitica
Figure 5.2.78 - Axillary Bubo
Figure 5.2.79 - Gram stain of Enterobacter
Figure 5.2.80 - Red pigment from Serratia on a piece of bread
Figure 5.2.81 - Warthin-Starry stain
Figure 5.2.82 - Gram stain of Legionella
Figure 5.2.83 - Silver stain of Legionella
Figure 5.2.84 - Buffered charcoal yeast extract agar
Figure 5.2.85 - Gram stain Campylobacter
Figure 5.2.86 - Gram stain of Vibrio cholerae
Figure 5.2.87 - “Rice-water” diarrhea
Figure 5.2.88 - Hematoxylin and eosin (H&E) stain of the gastric mucosa
Figure 5.2.89 - Silver based stain
Figure 5.2.90 - Stomach anatomy and physiology
Figure 5.2.91 - Acid-fast stain of Mycobacterium tuberculosis
Figure 5.2.92 - Lowenstein-Jensen medium
Figure 5.2.93 - Pathophysiology of Mycobacterium tuberculosis
Figure 5.2.94 - Miliary tuberculosis
Figure 5.2.95 - “Serpentine cords”
Figure 5.2.96 - PPD test
Figure 5.2.97 - Ghon complex
Figure 5.2.98 - Acid-fast stain of Mycobacterium leprae
Figure 5.2.99 - Pathophysiology of Mycobacterium leprae
Figure 5.2.100 - Leonine Facies
Figure 5.2.101 - Acid-fast stain of Mycobacterium avium-intracellulare
Figure 5.2.102 - Spirochetes via dark-field microscopy
Figure 5.2.103 -  Ixodes deer tick
Figure 5.2.104 - Giemsa stain of Borrelia
Figure 5.2.105 - Erythema migrans
Figure 5.2.106 - Bell’s palsy
Figure 5.2.107 - Electron microscopic image of Leptospira interrogans
Figure 5.2.108 - Dark-field photomicrograph of Treponema pallidum
Figure 5.2.109 - Chancre
Figure 5.2.110 - Maculopapular rash
Figure 5.2.111 - Maculopapular rash on the palms
Figure 5.2.112 - Condyloma lata
Figure 5.2.113 - Gumma
Figure 5.2.114 - Cross section of the spinal cord
Figure 5.2.115 - Notched teeth
Figure 5.2.116 - Saddle nose
Figure 5.2.117 - Rhagades
Figure 5.2.118 - Life cycle of Chlamydia trachomatis
Figure 5.2.119 - Giemsa stain of Chlamydia
Figure 5.2.120 -  Gram stain of Coxiella burnetii
Figure 5.2.121 -  Osler’s nodes
Figure 5.2.122 - Clue cells
Figure 5.2.123 - IgM antibodies
Figure 5.2.124 - Rocky Mountain Spotted Fever
Figure 5.2.125 - Weil Felix Test
Figure 5.2.126 - Ehrlichia within a monocyte
Figure 5.2.127 - Anaplasma within a granulocyte
Table 5.2.1 - Miscellaneous organisms
Figure 5.2.128 - Intestinal biopsy in Whipple disease
Figure 5.2.129 - Granuloma inguinale caused by Klebsiella granulomatis (male)
Figure 5.2.130 - Granuloma inguinale caused by Klebsiella granulomatis (female)
Figure 5.2.131 - Bipolar Donovan bodies from granuloma inguinale ulcer biopsy
Figure 5.3.1 - Fungi overview figure
Figure 5.3.2 - Histoplasma capsulatum yeast in macrophages
Figure 5.3.3 - Mold (left) and yeast (right)
Figure 5.3.4 - Verrucous skin lesion
Figure 5.3.5 - Squamous cell carcinoma
Figure 5.3.6 - Granulomatous nodule
Figure 5.3.7 - Broad-based budding by Blastomyces dermatitidis
Figure 5.3.8 - Endospore of Coccidioides immitis
Figure 5.3.9 - Coccidioides using the methenamine silver stain
Figure 5.3.10 - Erythema multiforme
Figure 5.3.11 - Erythema nodosum
Figure 5.3.12 - Budding cells of Paracoccidioides brasiliensis
Figure 5.3.13 - Hyphae and spores of a dermatophyte
Figure 5.3.14 - Tinea capitis
Figure 5.3.15 - Tinea corporis
Figure 5.3.16 - Tinea pedis
Figure 5.3.17 - Vesicular athlete’s foot
Figure 5.3.18 - Interdigital tinea pedis
Figure 5.3.19 - Onychomycosis
Figure 5.3.20 - Tinea cruris
Figure 5.3.21 - Tinea versicolor skin lesions
Figure 5.3.22 - Microscopic image of Malassezia furfur “Spaghetti and meatballs”
Figure 5.3.23 - Sporothrix schenckii (yeast form)
Figure 5.3.24 - Sporothrix schenckii (mold form)
Figure 5.3.25 - Sporothrix schenckii lesion along draining lymphatic channels
Figure 5.3.26 - Pseudohyphae and budding yeast
Figure 5.3.27 - Germ tube formation
Figure 5.3.28 - Oral thrush
Figure 5.3.29 - Esophageal candidiasis
Figure 5.3.30 - Vulvovaginitis caused by Candida albicans
Figure 5.3.31 - Microscopic morphology of Aspergillus fumigatus
Figure 5.3.32 - Aspergilloma (gross image)
Figure 5.3.33 - Aspergilloma (imaging)
Figure 5.3.34 - India ink stain
Figure 5.3.35 - Sabouraud dextrose agar
Figure 5.3.36 - Mucicarmine stain
Figure 5.3.37 - Microscopy of mucormycosis
Figure 5.3.38 - Silver stain of Pneumocystis jirovecii cysts
Figure 5.3.39 - Chest x-rays of Pneumocystis pneumonia with “ground glass” opacities
Figure 5.3.40 - Pneumatoceles
Figure 5.3.41 - Fluorescent antibody stain
Figure 5.4.1 - Parasites overview figure
Figure 5.4.2 - Giardia lamblia cyst
Figure 5.4.3 - Giardia lamblia trophozoite
Figure 5.4.4 - Red blood cells within Entamoeba cytoplasm (erythrophagocytosis)
Figure 5.4.5 - Entamoeba egg in stool sample
Figure 5.4.6 - Cryptosporidium oocysts with acid-fast staining
Figure 5.4.7 - Trypomastigotes on peripheral blood smear
Figure 5.4.8 - Malaria Life Cycle
Figure 5.4.9 - Plasmodium trophozoite ring form within red blood cells
Figure 5.4.10 - Babesia trophozoite ring form (“Maltese cross”) within red blood cells
Figure 5.4.11 - Barium esophagram showing achalasia
Figure 5.4.12 - Cutaneous leishmaniasis
Figure 5.4.13 - Flagellated Gardnerella trophozoites
Figure 5.4.14 - Gardnerella vaginalis on saline microscopy
Figure 5.4.15 - Pinworm egg
Figure 5.4.16 - Life cycle of parasites that cause Löffler syndrome
Figure 5.4.17 - Ascaris lumbricoides egg
Figure 5.4.18 - Strongyloides stercoralis larva
Figure 5.4.19 - Hookworm egg
Figure 5.4.20 - Cutaneous hookworm lesion
Figure 5.4.21 - Striated muscle tissue infected with Trichinella spiralis
Figure 5.4.22 - Periorbital edema
Figure 5.4.23 - Whipworm egg
Figure 5.4.24 - Blackfly
Figure 5.4.25 - Elephantiasis
Figure 5.4.26 - Taenia solium life cycle
Figure 5.4.27 - Taenia solium on microscopy
Figure 5.4.28 - Taenia solium egg in stool sample
Figure 5.4.29 - Neurocysticercosis on imaging
Figure 5.4.30 - Echinococcus granulosus life cycle
Figure 5.4.31 - Echinococcus granulosus tapeworm on microscopy
Figure 5.4.32 - Liver hydatid cyst on MRI
Figure 5.4.33 - Liver hydatid cyst (gross)
Figure 5.4.34 - Schistosoma life cycle
Figure 5.4.35 - Lateral spine on Schistosoma egg (mansoni or japanicom)
Figure 5.4.36 - Lateral spine on Schistosoma haematobium egg
Figure 5.4.37 - Scabies-related erythematous papules
Figure 5.4.38 - Pediculus capitis louse
Figure 5.4.39 - Pediculus nit on hair shaft
Figure 5.4.40 - Pediculus pubis louse
Figure 5.4.41 - Pediculus corporis louse
Figure 5.4.42 - Virus overview figure
Figure 5.4.43 - Trigeminal ganglion
Figure 5.4.44 - Gingivostomatitis
Figure 5.4.45 - Viral keratoconjunctivitis
Figure 5.4.46 - Herpetic whitlow
Figure 5.4.47 - Herpes labialis
Figure 5.4.48 - Erythema multiforme
Figure 5.4.49 - HSV-related esophageal ulcers
Figure 5.4.50 - MRI of temporal lobe encephalitis
Figure 5.4.51 - Neonatal herpes
Figure 5.4.52 - Vesicular and ulcerative herpes lesions
Figure 5.4.53 - Multinucleated giant cells on Tzanck smear
Figure 5.4.54 - Cowdry bodies in hepatocytes
Figure 5.4.55 - Chickenpox
Figure 5.4.56 - Shingles
Figure 5.4.57 - Herpes zoster ophthalmicus
Figure 5.4.58 - Cytotoxic T-cells on EBV-related peripheral blood smear
Figure 5.4.59 - Starry sky appearance on biopsy of Burkitt’s lymphoma
Figure 5.4.60 - MRI showing ring-enhancing lesion
Figure 5.4.61 - CMV retinitis
Figure 5.4.62 - “Blueberry muffin” rash
Figure 5.4.63 - Periventricular calcifications on CT imaging
Figure 5.4.64 - Owl’s eye appearance on lung biopsy
Figure 5.4.65 - Skin lesions in Kaposi’s sarcoma
Figure 5.4.66 - HBV structure
Table 5.4.1 - Descriptions of serologic markers in Hepatitis B
Figure 5.4.67 - Serological markers in acute hepatitis B infection
Figure 5.4.68 - Serological markers in chronic hepatitis B infection
Figure 5.4.69 - H&E stain showing ground glass appearance
Figure 5.4.70 - Reverse transcriptase in HBV
Figure 5.4.71 - Smallpox
Figure 5.4.72 - Umbilicated lesions in Molluscum contagiosum
Figure 5.4.73 - Plantar warts
Figure 5.4.74 - Anal warts
Figure 5.4.75 - HPV carcinogenesis mechanism
Figure 5.4.75 - Koilocytes on Pap smear
Figure 5.4.77 - Progressive multifocal leukoencephalopathy (PML) on MRI
Figure 5.4.78 - Fifth disease
Figure 5.4.79 - HIV structure
Figure 5.4.80 - HIV replication cycle
Figure 5.4.81 - Time course of HIV infection
Figure 5.4.82 - HIV diagnosis
Figure 5.4.83 - Fat and lymphoid aggregates seen on HCV liver biopsy
Figure 5.4.84 - Lichen planus
Figure 5.4.85 - Maculopapular rash of Rubella
Figure 5.4.86 - Chikungunya-related erythematous maculopapular rash
Figure 5.4.87 - Genetic shift and genetic drift
Figure 5.4.88 - Acute HDV-HBV coinfection
Figure 5.4.89 - HDV superinfection
Figure 5.4.90 - Steeple sign on radiograph in croup
Figure 5.4.91 - Bronchiolitis on chest radiograph
Figure 5.4.92 - Maculopapular rash in measles
Figure 5.4.93 - Koplik’s spots
Figure 5.4.94 - Giant cells
Figure 5.4.95 - Mumps-related parotitis
Figure 5.4.96 - Rabies virus structure
Figure 5.4.97 - Rabies virus bullet-shaped envelopes on electron microscopy
Figure 5.4.98 - Rabies virus Negri bodies
Figure 5.4.99 - Retrograde movement of Rabies virus
Figure 5.4.100 - Positive-sense and negative-sense viruses
Figure 5.4.101 - Transmission electron micrograph of Rotavirus
Figure 5.4.102 - Coxsackie-related palmar vesicular lesions
Figure 5.4.103 - Coxsackie-related plantar vesicular lesions
Figure 5.4.104 - Coxsackie-related oropharyngeal vesicular lesions (herpangina)
Figure 5.4.105 - Councilman bodies in HAV

Citation preview

MICROBIOLOGY

MEDICAL COURSE AND STEP 1 REVIEW FIRST EDITION Accompanies online videos taught by Michael Christensen & Rhett Thomson physeo.com

Copyright © 2018 by Physeo All rights reserved. No part of this publication may be reproduced, distributed, or transmitted in any form or by any means, including photocopying, recording, or other electronic or mechanical methods, without the prior written permission of Physeo, except in the case of personal study purposes.

TABLE OF CONTENTS MICROBIOLOGY FUNDAMENTALS................................................................................6 Section 1 - Bacterial Structures...................................................................................................................................................... 6 Section 2 - Virulence.................................................................................................................................................................... 10 Section 3 - Bacterial Genetics...................................................................................................................................................... 20 Section 4 - Prion Diseases............................................................................................................................................................ 27 Section 5 - Introduction to Fungi................................................................................................................................................. 29 Section 6 - Viral Structures.......................................................................................................................................................... 32 Section 7 - Viral Genetics............................................................................................................................................................ 35 Section 8 - Viral Genomes and Replication................................................................................................................................. 38

BACTERIA.............................................................................................................................41 Section 1 - Clostridium tetani...................................................................................................................................................... 42 Section 2 - Clostridium botulinum............................................................................................................................................... 44 Section 3 - Clostridium difficile .................................................................................................................................................. 46 Section 4 - Clostridium perfringens............................................................................................................................................. 48 Section 5 - Bacillus anthracis...................................................................................................................................................... 50 Section 6 - Bacillus cereus........................................................................................................................................................... 52 Section 7 - Listeria monocytogenes............................................................................................................................................. 54 Section 8 - Corynebacterium diphtheriae ................................................................................................................................... 56 Section 9 - Viridans streptococci................................................................................................................................................. 58 Section 10 - Streptococcus pneumoniae...................................................................................................................................... 60 Section 11 - Streptococcus pyogenes........................................................................................................................................... 62 Section 12 - Streptococcus agalactiae......................................................................................................................................... 66 Section 13 - Streptococcus gallolyticus....................................................................................................................................... 68 Section 14 - Enterococci.............................................................................................................................................................. 70 Section 15 - Staphylococcus aureus............................................................................................................................................. 72 Section 16 - Staphylococcus epidermidis.................................................................................................................................... 74 Section 17 - Staphylococcus saprophyticus................................................................................................................................. 76 Section 18 - Nocardia ................................................................................................................................................................. 78 Section 19 - Actinomyces israelii................................................................................................................................................. 80 Section 20 - Neisseria overview ................................................................................................................................................. 82 Section 21 - Neisseria meningitidis............................................................................................................................................. 84 Section 22 - Neisseria gonorrhoeae ........................................................................................................................................... 86 Section 23 - Moraxella catarrhalis ............................................................................................................................................. 88 Section 24 - Haemophilus influenzae........................................................................................................................................... 90 Section 25 - Bordetella pertussis................................................................................................................................................. 94 Section 26 - Pasteurella multocida, Brucella spp., and Francisella tularensis........................................................................... 96 Section 27 - Salmonella spp......................................................................................................................................................... 98 Section 28 - Salmonella typhi.................................................................................................................................................... 100 Section 29 - Shigella.................................................................................................................................................................. 102 Section 30 - E. coli..................................................................................................................................................................... 104 Section 31 - Klebsiella............................................................................................................................................................... 108 Section 32 - Proteus mirabilis................................................................................................................................................... 110 Section 33 - Pseudomonas aeruginosa...................................................................................................................................... 112 Section 34 - Yersinia pestis and enterocolitica.......................................................................................................................... 116 Section 35 - Enterobacter, Citrobacter, & Serratia marcescens................................................................................................ 118 Section 36 - Bartonella henselae .............................................................................................................................................. 120 Section 37 - Legionella pneumophila........................................................................................................................................ 122 Section 38 - Campylobacter jejuni............................................................................................................................................ 124 Section 39 - Vibrio cholerae & Vibrio vulnificus....................................................................................................................... 126 Section 40 - Helicobacter pylori................................................................................................................................................ 128 Section 41 - Mycobacterium tuberculosis.................................................................................................................................. 130 Section 42 - Mycobacterium leprae .......................................................................................................................................... 134

Section 43 - Mycobacterium avium-intracellulare, Scrofulaceum, and Marinum.................................................................... 138 Section 44 - Borrelia burgdorferi.............................................................................................................................................. 140 Section 45 - Leptospira interrogans........................................................................................................................................... 142 Section 46 - Treponema pallidum.............................................................................................................................................. 144 Section 47 - Chlamydia trachomatis, Chlamydophila pneumoniae, and Chlamydophila psittaci............................................ 148 Section 48 - Coxiella burnetii.................................................................................................................................................... 150 Section 49 - Gardnerella vaginalis............................................................................................................................................ 152 Section 50 - Mycoplasma pneumoniae...................................................................................................................................... 154 Section 51 - Rickettsia prowazekii, Rickettsia rickettsii, and Rickettsia typhi.......................................................................... 156 Section 52 - Ehrlichia & Anaplasma......................................................................................................................................... 158 Section 53 - Miscellaneous Organisms...................................................................................................................................... 160

FUNGI...................................................................................................................................165 Section 1 - Histoplasma capsulatum.......................................................................................................................................... 166 Section 2 - Blastomyces dermatitidis......................................................................................................................................... 168 Section 3 - Coccidioides immitis................................................................................................................................................ 172 Section 4 - Paracoccidioides brasiliensis.................................................................................................................................. 174 Section 5 - Microsporum, Trichophyton, and Epidermophyton................................................................................................. 176 Section 6 - Malassezia furfur..................................................................................................................................................... 180 Section 7 - Sporothrix schenckii................................................................................................................................................ 182 Section 8 - Candida albicans..................................................................................................................................................... 184 Section 9 - Aspergillus fumigatus.............................................................................................................................................. 188 Section 10 - Cryptococcus neoformans..................................................................................................................................... 190 Section 11 - Mucor & Rhizopus spp.......................................................................................................................................... 192 Section 12 - Pneumocystis jirovecii........................................................................................................................................... 194

PARASITES.........................................................................................................................197 Section 1 - Giardia lamblia....................................................................................................................................................... 198 Section 2 - Entamoeba histolytica............................................................................................................................................. 200 Section 3 - Cryptosporidium...................................................................................................................................................... 202 Section 4 - Toxoplasma gondii................................................................................................................................................... 204 Section 5 - Naegleria fowleri..................................................................................................................................................... 206 Section 6 - Trypanosoma brucei, gambiense, and rhodesiense................................................................................................. 208 Section 7 - Malaria.................................................................................................................................................................... 210 Section 8 - Babesia.................................................................................................................................................................... 214 Section 9 - Trypanosoma cruzi.................................................................................................................................................. 216 Section 10 - Leishmania donovani............................................................................................................................................. 218 Section 11 - Trichomonas vaginalis........................................................................................................................................... 220 Section 12 - Enterobius vermicularis (pinworm)...................................................................................................................... 222 Section 13 - Ascaris lumbricoides (giant roundworm).............................................................................................................. 224 Section 14 - Strongyloides stercoralis (threadworm)................................................................................................................ 226 Section 15 - Ancylostoma duodenale and Necator americanus (hookworms).......................................................................... 228 Section 16 - Trichinella spiralis................................................................................................................................................. 230 Section 17 - Trichuris trichiura (whipworm)............................................................................................................................ 232 Section 18 - Toxocara canis....................................................................................................................................................... 234 Section 19 - Onchocerca volvulus............................................................................................................................................. 236 Section 20 - Loa loa and Wuchereria bancrofti......................................................................................................................... 238 Section 21 - Taenia solium......................................................................................................................................................... 240 Section 22 - Diphyllobothrium latum........................................................................................................................................ 243 Section 23 - Echinococcus granulosus...................................................................................................................................... 244 Section 24 - Schistosoma haematobium, mansoni, and japanicom........................................................................................... 246 Section 25 - Clonorchis sinensis................................................................................................................................................ 248 Section 26 - Sarcoptes scabiei................................................................................................................................................... 250

Section 27 - Pediculus corporis, pubis and capitis.................................................................................................................... 252

VIRUSES..............................................................................................................................254 Section 1 - Herpes simplex virus-1 (HSV-1) and herpes simplex virus-2 (HSV-2).................................................................. 255 Section 2 - Varicella-zoster virus (VZV)................................................................................................................................... 260 Section 3 - Epstein-Barr Virus (EBV)....................................................................................................................................... 262 Section 4 - Cytomegalovirus (CMV)......................................................................................................................................... 264 Section 5 - Roseola (HHV-6 and HHV-7).................................................................................................................................. 266 Section 6 - Kaposi sarcoma (HHV-8)........................................................................................................................................ 268 Section 7 - Hepatitis B Virus (HBV)......................................................................................................................................... 270 Section 8 - Smallpox, Cowpox, and Molluscum contagiosum.................................................................................................. 274 Section 9 - Human papillomavirus............................................................................................................................................ 276 Section 10 - Adenovirus............................................................................................................................................................. 280 Section 11 - JC Virus and BK Virus........................................................................................................................................... 282 Section 12 - Parvovirus B19...................................................................................................................................................... 284 Section 13 - Coronavirus and SARS.......................................................................................................................................... 286 Section 14 - Human immunodeficiency virus (HIV)................................................................................................................. 288 Section 15 - Human T-lymphotropic virus (HTLV)................................................................................................................... 292 Section 16 - Hepatitis C Virus (HCV)....................................................................................................................................... 294 Section 17 - Yellow fever .......................................................................................................................................................... 296 Section 18 - Dengue Fever......................................................................................................................................................... 298 Section 19 - West Nile Virus and St. Louis Encephalitis........................................................................................................... 300 Section 20 - Zika Virus.............................................................................................................................................................. 302 Section 21 - Rubella................................................................................................................................................................... 304 Section 22 - Eastern, Western, and Venezuelan equine encephalitis viruses; Chikungunya virus............................................ 306 Section 23 - California Encephalitis.......................................................................................................................................... 308 Section 24 - Rift Valley Fever and Crimean-Congo Hemorrhagic Fever Virus........................................................................ 310 Section 25 - Hantavirus.............................................................................................................................................................. 312 Section 26 - Influenza virus....................................................................................................................................................... 314 Section 27 - Lassa fever encephalitis and Lymphocytic choriomeningitis virus....................................................................... 316 Section 28 - Hepatitis D Virus................................................................................................................................................... 317 Section 29 - Parainfluenza Virus................................................................................................................................................ 320 Section 30 - RSV and hMPV..................................................................................................................................................... 322 Section 31 - Measles.................................................................................................................................................................. 324 Section 32 - Mumps................................................................................................................................................................... 326 Section 33 - Rabies Virus........................................................................................................................................................... 328 Section 34 - Ebola Virus and Marburg Virus............................................................................................................................. 332 Section 35 - Norovirus............................................................................................................................................................... 334 Section 36 - Hepatitis E virus.................................................................................................................................................... 336 Section 37 - Rotavirus and Colorado Tick Fever....................................................................................................................... 338 Section 38 - Poliovirus............................................................................................................................................................... 340 Section 39 - Echovirus............................................................................................................................................................... 342 Section 40 - Rhinovirus............................................................................................................................................................. 344 Section 41 - Coxsackieviruses (A & B)..................................................................................................................................... 346 Section 42 - Hepatitis A Virus.................................................................................................................................................... 348

We would like to extend a special thanks to the following individuasl who have spent many hours tutoring, guiding and consulting this work, making Physeo Anatomy possible. Paloma F Cariello, MD, MPH Assistant Professor Division of Infectious Diseases University of Utah School of Medicine Salt Lake City, UT Karen Eilbeck, Ph.D. Professor Biomedical Informatics University of Utah Vicente Planelles, Ph.D. Professor Division of Microbiology and Immunology Department of Pathology University of Utah School of Medicine Salt Lake City, UT

6

MICROBIOLOGY FUNDAMENTALS Section 1 - Bacterial Structures I. There are several bacterial structures to be familiar with for board exams. These structures can help bacteria infect host cells and survive. Some structures are unique to certain groups and can be useful in distinguishing bacterial organisms.

Structure Cytoplasmic membrane

Description • • •



Peptidoglycan that provides rigidity and structural support to cell Thick in gram-positive bacteria (gram staining with crystal violet causes blue appearance) Thin in gram-negative bacteria Absent in Mycoplasma species which have added sterols to maintain stability (must grow on Eaton’s agar) Contains mycolic acid in Mycobacteria species (must use acid-fast stain)

Flagella



Provides bacterial motility

Pilus (fimbria)

• •

Allows bacteria to bind to host cells Allows bacteria-bacteria binding during conjugation

• •

Network of polysaccharides Slime layer: disorganized and loosely attached to cell wall → creates biofilm on prosthetics (eg, S. epidermidis) Capsule: organized and firmly attached to cell wall → protects from phagocytosis (targeted by many vaccines) Note: the capsule of Bacillus anthracis is made of a protein network of poly-ᴅ-glutamate

Cell wall

Glycocalyx (capsule or slime layer)

• • • •

Phospholipid bilayer Contains lipoteichoic acids in gram-positive bacteria → stimulates host immune response via TNF-ɑ and IL-1 release Contains penicillin-binding proteins (PBPs)

• • •

Outer membrane

Periplasm

Endospore

• • •

Only present in gram-negative organisms (gram counter-staining with safranin causes red appearance) Contains porins → transfer of nutrients Contains lipopolysaccharides (LPS: lipid A, core polysaccharide, and O antigen) Lipid A stimulates host immune response via TNF and IL-1 release → fever, shock, diarrhea

• •

Located between outer membrane and cytoplasmic membrane in gram-negative bacteria Contains hydrolytic enzymes (eg, β-lactamases)



Layer formed during dormant state → resists heat, chemicals, dehydration and starvation → survival of bacteria Only present in gram-positive organisms (Clostridium and Bacillus) Center containing DNA and dipicolinic acid (DPA) → cytoplasmic membrane (spore core) → peptidoglycan wall (spore cortex) → cytoplasmic membrane → keratin-like protein wall (spore coat) → exosporium (present only in B. anthracis and B. cereus)

• •

Table 5.1.1 - Bacterial structures

7

Figure 5.1.1 - Gram positive bacterial envelope

Figure 5.1.2 - Gram negative bacterial envelope

8

Figure 5.1.3 - Envelope of Mycoplasma spp.

Figure 5.1.4 - Envelope of Mycobacteria spp.

Figure 5.1.5 - Acid-fast stain of Mycobacterium Tuberculosis

9

Figure 5.1.6 - Bacterial endospore A. Polymerases in bacteria 1. DNA polymerase (replication → more DNA) a) Mistakes can harm progeny 2. RNA polymerase (transcription → mRNA) a) Mistakes do not harm progeny

?

REVIEW QUESTIONS 1. A researcher is attempting to create a novel staining agent that will only bind to certain bacterial species. This species cannot form an endospore, does not contain mycolic acids, and contains only one phospholipid bilayer. Which of the following does the organism described most likely possess? A. A cytokine inducing structure in outer membrane B. A layer that tightly binds to the safranin counterstain C. A protein structure allowing for motility D. A relatively thin layer of peptidoglycan

• Correct answer: C • The question stem describes a gram positive organism • Contains only one phospholipid bilayer, so its not gram negative • Does not contain mycolic acids, so its not mycobacteria • Gram positive organisms may contain flagella • Choice A is incorrect because this describes the endotoxin of gram negative bacteria • Choice B is incorrect because Safranin binds to the outer membrane found only in gram negative bacteria • Choice D is incorrect because thin peptidoglycan layers are found in Mycobacteria and gram negative organisms, not in gram positive bacteria

10 Section 2 - Virulence I. Virulence describes the bacteria’s ability to do 3 things: A. Invade the host B. Evade immune defenses C. Inflict disease

Virulence Factor

Action

Role in Virulence

Pili

Adherence

Invade the host

IgA protease

Adherence

Invade the host

Flagella

Motility

Invade the host

Capsule

Prevent phagocytosis

Evade immune defenses

M protein

Prevent phagocytosis

Evade immune defenses

Protein A

Prevent phagocytosis

Evade immune defenses

Type III secretion system

Direct injection of exotoxins into host cell

Inflict disease

Endotoxins

Triggers host immune response

Inflict disease

Exotoxins

Secreted from bacteria (activities vary)

Inflict disease

Table 5.1.2 - Overview of virulence factors II. IgA protease A. Cleaves IgA antibodies B. Allows bacteria to adhere to mucosal surfaces C. Secreted by multiple bacteria: 1. S. pneumoniae 2. H. influenzae type b 3. N. meningitidis 4. N. gonorrhoeae III. M protein A. Prevents opsonization and phagocytosis B. Similar to antigens present in the heart → antibodies that form against the M protein can attack the heart as well → rheumatic fever C. Utilized by S. pyogenes (GAS)

IV. Protein A A. Binds to the Fc portion of IgG antibodies → only the Fab portion of IgG is exposed B. Prevents opsonization and phagocytosis C. On the surface of S. aureus

11

Figure 5.1.7 - Protein A mechanism V. Type III secretion system A. Protein structure that can insert into host cells → injects exotoxins directly into host cell B. Utilized by several gram negative organisms: 1. Salmonella spp 2. Shigella spp 3. E. coli spp 4. P. aeruginosa

12

Photo Credit: (left) Pixie [CC BY-SA 3.0 (https://creativecommons.org/license/by-sa/3.0)]; (right) Schraidt O. Lefebre MD, Brunner MJ, Schmied WH, Schmidt A, Radics J, Mechtier K, Galán JE, Marlovits TC [CC BY 2.5 (https://creativecommons.org/license/by-sa/2.5)]

Figure 5.1.8 - Visual demonstration of Type III secretion systems VI. Endotoxins A. Lipopolysaccharide (LPS) on the surface of gram negative bacteria B. Lipid A is the toxic component of LPS C. Bacterial cell death → release of lipid A → host release of cytokines (TNF-α, IL-1, and IL-6) → sepsis → severe sepsis → septic shock D. Source of endotoxin genes: 1. Bacterial chromosome VII. Sepsis A. Sepsis: “Life-threatening organ dysfunction caused by a dysregulated host response to infection” (2016 SCCM/ESICM task force) B. Septic shock: vasopressor required (2016 SCCM/ ESICM task force) C. Virulence factor triggers host immune response → organ dysfunction (hypotension) 1. Gram negative bacteria (lipid A) 2. Gram positive bacteria 3. Fungi

VIII. Exotoxins A. Secreted proteins that inflict disease B. Utilized by most gram positive and some gram negative bacteria C. Source of exotoxin genes: 1. Bacterial chromosome 2. Plasmids 3. Lysogenic bacteriophages following lysogenic conversion (phage DNA can produce protein products)

13 Exotoxin

Mechanism

Result

Diphtheria toxin (C. diphtheriae)

Inhibit elongation factor 2 (EF-2)

Blocked protein synthesis

Exotoxin A (P. aeruginosa)

Inhibit elongation factor 2 (EF-2)

Blocked protein synthesis

Shiga toxin (Shigella spp)

Remove adenine from rRNA on 60S ribosome (invades GI cells)

Blocked protein synthesis

Shiga-like toxin (EHEC)

Remove adenine from rRNA on 60S ribosome (does not invade GI cells)

Blocked protein synthesis

Heat-labile toxin (LT) (ETEC)

Stimulates Gαs subunit → ↑ adenylate cyclase → ↑cAMP → ↑ Cl- & H2O secreted in gut

Watery diarrhea

Heat-stable toxin (ST) (ETEC)

Guanylate cyclase overactivation → ↑ cGMP → ↓ Cland H2O reabsorption in the gut

Watery diarrhea

Cholera toxin (V. cholera)

Stimulates Gαs subunit → ↑ adenylate cyclase → ↑cAMP → ↑ Cl- & H2O secreted in gut

Watery diarrhea

Edema factor (B. anthracis)

Mimics adenylate cyclase → ↑ cAMP → cutaneous edema

Edema around black eschar

Pertussis toxin (B. pertussis)

Blocks Gαi subunit → ↑ adenylate cyclase → ↑ cAMP → PMN chemotaxis impaired

Persistence of microbe

Tetanospasmin (C. tetani)

Cleaves SNARE proteins → lack of inhibitory neurotransmitters (GABA, glycine)

↑ muscular tone

Botulinum toxin (C. botulinum)

Cleaves SNARE proteins → lack of stimulatory neurotransmitter (ACh)

↓ muscular tone

Alpha toxin (C. perfringens)

Lecithinase (a phospholipase) that breaks down cell membrane

Hemolysis and myonecrosis

Streptolysin O (S. pyogenes)

A protein that breaks down cell membrane

Hemolysis

Toxic shock syndrome toxin (TSST-1) (S. aureus)

A superantigen that strongly binds the MHC class II antigen-presenting cells to the CD4 on TH1 cells → perpetual release of IFN-γ, IL-1, IL-2, and TNF-α

Toxic shock syndrome (fever, rash, shock)

Erythrogenic toxin A (S. pyogenes)

A superantigen that strongly binds the MHC class II antigen-presenting cells to the CD4 on TH1 cells → perpetual release of IFN-γ, IL-1, IL-2, and TNF-α

Toxic shock-like syndrome (fever, rash, shock) & scarlet fever

Table 5.1.3 - Bacterial exotoxins

14

Figure 5.1.9 - Exotoxins that inhibit protein synthesis

Figure 5.1.10 - Exotoxins that increase fluid secretion using adenylate cyclase

15

Figure 5.1.11 - Exotoxin (heat-stable toxin) that increases fluid secretion through guanylate cyclase

Figure 5.1.12 - Pertussis toxin mechanism

16

Figure 5.1.13 - Mechanism of tetanospasmin

Figure 5.1.14 - Mechanism of botulinum toxin

17

Figure 5.1.15 - Mechanism of exotoxins that disrupt cell membranes

Figure 5.1.16 - Mechanism of superantigens

18

?

REVIEW QUESTIONS 1. A bacterial organism produces a virulence factor that interacts with host antibodies, allowing it to adhere to host surfaces. Which of the following statements is consistent with this virulence factor? A. B. C. D.

It is produced by S. aureus It is secreted from the pathogen It can lead to rheumatic fever It is part of the cell surface

• Correct answer: B • The virulence factor in question is IgA protease, based on the following: • It interacts with host antibodies • The above interaction allows the bacteria to adhere to mucosal surfaces • IgA protease is secreted • Choice A is incorrect because S. aureus produces protein A, not IgA protease • Choice C is incorrect because this describes the M protein of S. pyogenes • Choice D is incorrect because IgA protease is not part of the cell surface

2. An 18-year-old female who has been “feeling sick” is brought to the emergency department by her mother. Upon arrival, her temperature is 39.4°C (103.0°F) and her blood pressure is 78/45 mmHg. Blood cultures confirm bacteremia with S. pyogenes. Which statement is true regarding how the pathogen caused her septic shock? A. The organism released a large quantity of cytokines B. Lipid A dysregulated the immune response C. A virulence factor triggered the release of TNF-α D. An exotoxin caused direct cellular damage • Correct answer: C • This presentation is consistent with sepsis based on the following: • Bacteremia • Fever • Hypotension • S. pyogenes is a gram positive organism and contains many virulence factors which can trigger host release of cytokines, including TNF-α • Since the question stem states the patient has “septic shock”, it should be assumed the patient requires a vasopressor • Choice A is incorrect because this implies that S. pyogenes directly releases cytokines • Choice B is incorrect because lipid A is found only on gram negative bacteria • Choice D is incorrect because cellular damage does not cause sepsis (host response to virulence factors cause sepsis)

19

?

REVIEW QUESTIONS 3. A 30-year-old man with bloody diarrhea is diagnosed with a Shigella infection. Which statement describes the mechanism through which Shiga toxin alters host cell activities? A. B. C. D.

Inhibition of an elongation factor Gαs subunit stimulation Ribonucleotide removal on rRNA Increased levels of cAMP

• Correct answer: C • Shiga toxin and Shiga-like toxins both inhibit protein synthesis by removing adenine (a ribonucleotide) from the rRNA of the 60S ribosomal subunit • Choice A is incorrect because this describes the mechanism of diphtheria toxin and exotoxin A from Pseudomonas aeruginosa • Choices B and D are incorrect because these describe the mechanism by which cholera toxin and the heat-labile toxin (HT) of ETEC cause watery diarrhea

4. A 15-year-old male is infected with a bacterial organism that releases an exotoxin. The role of this exotoxin is to prevent the release of glycine in the synaptic cleft of neurons. This describes which exotoxin? A. B. C. D. E.

Alpha toxin Tetanospasmin Botulinum toxin Streptolysin O Erythrogenic exotoxin A

• Correct answer: B • Glycine and GABA (γ-aminobutyric acid) are inhibitory neurotransmitters • Tetanospasmin prevents the release of these neurotransmitters, leading to motor disinhibition • Choices A and D are incorrect because these exotoxins disrupt cell membranes and cause myonecrosis and necrotizing fasciitis • Choice C is incorrect because botulinum toxin prevents the release of ACh, leading to paralysis • Choice E is incorrect because is a superantigen that causes shock

20 Section 3 - Bacterial Genetics I. Genetic mutations, alterations, and exchange are significant because these activities can increase the virulence of bacteria.

Genetic Change

Direction

Speed

Description

Significance

Random mutation

Vertical

Slow



Genes randomly mutated over time

• •

Increased virulence Antibiotic resistance

Phase and antigenic variation

Vertical

Fast



Genes are randomly turned on or off within a bacterium → antigenic or structural changes (e.g. change to capsule, loss of pili)



Increased virulence



Naked bacterial DNA enters cytoplasm of competent bacteria (S. pneumoniae, N. meningitidis, H. influenzae type b) Donor DNA is incorporated into recipient chromosome

• •

Increased virulence Utilized often in laboratory research

F+ cell transfers plasmid to F─ cell → recipient cell becomes F+ cell Plasmid integrates into chromosome → Hfr cell → some genes copied and transferred → recipient becomes recombinant F─ cell Hfr cell excises plasmid genes → novel plasmid transferred → recipient becomes F’ cell

• •

Increased virulence Antibiotic resistance

Bacteriophage injects viral DNA into host → viral DNA replicates and breaks up the host chromosome (lytic phage) Viral capsids are formed and inadvertently include host DNA → bacteria is destroyed and viral progeny released (lytic cycle) Viral progeny find new bacterial host and inject newly-acquired bacterial DNA



Increased virulence



Increased virulence (S. pyogenes erythrogenic toxin, Botulinum toxin, Cholera toxin, Diphtheria toxin, Shiga toxin) Antibiotic resistance

Transformation

Horizontal

Fast

• •

Conjugation

Horizontal

Fast

• • •

Generalized transduction

Horizontal

Fast

• • •

Specialized transduction

• Horizontal

Fast •

Bacteriophage injects viral DNA into host → viral DNA is incorporated into the host chromosome and remains dormant (lysogenic phage; prophage) When triggered, viral DNA is excised from the host chromosome, inadvertently excising bacterial genes → viral DNA replicates and lyses the host cell (lytic phage) Lysogenic conversion: When the prophage DNA produces gene products that enhance the virulence of the host bacteria (exotoxins) before entering the lytic cycle

Table 5.1.4 - Mechanisms of genetic change in bacteria



21

Figure 5.1.19 - Transformation in bacteria

Figure 5.1.17 - Random mutation in bacteria

Figure 5.1.18 - Phase variation in bacteria

22

Figure 5.1.20 - Conjugation in bacteria

23

Figure 5.1.21 - Generalized transduction (lytic cycle)

24

Figure 5.1.22 - Specialized transduction (lysogenic cycle) II. Restriction modification system A. Bacteria produce “DNAases” (eg, restriction endonucleases) to degrade invading viral DNA → foreign DNA cannot be incorporated into host genome B. Bacteria methylate their own DNA at certain regions to protect it from enzyme degradation C. Bacteria can destroy themselves if methylation fails

25

Figure 5.1.23 - Restriction modification III. Transposable elements A. DNA segments (transposons) can excise themselves and integrate somewhere else 1. From plasmid integrates into bacterial chromosome 2. From bacterial chromosome into plasmid 3. From bacterial chromosome to another part of the chromosome B. Create plasmids resistant to multiple drugs I. Loss of virulence A. Selective pressures → bacteria with advantageous genes become more abundant → increased virulence

B. Lack of selective pressures → genes (e.g. plasmids) that confer virulence are not advantageous → decreased virulence

26

?

REVIEW QUESTIONS 1. A colony of F─ bacteria is introduced to a novel culture medium in the absence of other competing bacteria. After a period of time, the bacteria is examined from two different areas of the culture medium. The bacteria from one area (A) has produced a slime layer of polysaccharides that was not initially present. Bacteria from the second area (B) has not produced this slime layer, and are less abundant than the bacteria in area B. The researchers conclude that the slime layer was initially produced by one bacterium in the first generation. Which of the following processes describes how bacteria in area A began producing the slime layer? A. B. C. D. E.

Transformation Specialized transduction Phase variation Conjugation Random mutation

• Correct answer: C • The question stem describes phase variation: • Within the first generation, a single bacterium started producing a slime layer which it had not produced earlier in its life → the change was sudden, not gradual over generations → genes must have been turned on

• Choices A and D are incorrect because there were no other bacterial species within the growth medium from which gain new genes through transformation or conjugation • Choice B is incorrect because there is no evidence of bacteriophages within the medium • Choice E is incorrect because random mutations that would lead to a slime layer would’ve been present in the bacteria from the start of its life, not suddenly part way through its life

27 Section 4 - Prion Diseases

1. Prying: prion diseases 2. Sponge character holding a brain: spongiform encephalopathy 3. Brain juices (CSF) pooling in the shape of “14-3-3”: 14-3-3 proteins are found in CSF of infected patients 4. Punching the fish with the scissors: prions are protease resistant 5. Helical weapon knocking fish into hanging sheets: pathologic prions convert normal prion proteins from an α-helical configuration (PrPc) to a β-pleated sheet formation (PrPsc) 6. Fish tripping into sheets and getting other fish tangled in sheets: β-pleated sheet proteins convert other proteins into more β-pleated sheets

7. Taxi: causes ataxia 8. Demon shouting: causes dementia 9. Crab man run over and killed: causes death 10. Sea-cows hanging outside butcher shop: can be caused by “Mad cow disease” (bovine spongiform encephalopathy) 11. Squid family with insomnia and one dead: can be caused by fatal familial insomnia 12. A cross in a field: can be caused by Creutzfeld-Jakob disease (CJD) 13. Snail eating the deceased snail: can be acquired via Kuru (tribal cannibalism) 14. Chained up autoclave: prions are autoclave-resistant

28 REVIEW QUESTIONS

?

1. A 65-year-old male with dementia passes away. It is reported that he had difficulty walking and experienced frequent falls. His family requests that an autopsy be performed in order to provide a definitive diagnosis. A biopsy of his brain is obtained and shown below.

Photo Credit: Jensflorian [CC BY-SA 3.0 (https://creativecommons.org/ licenses/by-sa/3.0)]

Figure 5.1.24 - Brain biopsy with vacuolations from spongiform encephalopathy

Photo Credit: CDC (public domain) via Wikimedia Commons

• Which of the following describes the proteins responsible for neuronal cell death in this patient’s brain? A. B. C. D.

PrPc; α-helical PrPc; β-sheet PrPsc; α-helical PrPsc; β-sheet

• Correct answer: D • The patient has spongiform encephalopathy based on the following: • Ataxia • Dementia • Vacuolations in brain biopsy • Pathologic prions (PrPsc) are in the β-sheet configuration • Choice A is incorrect because pathologic prions are not in the α-helical configuration and PrPC describes normal prions • Choice B is incorrect because PrPC describes normal prions • Choice C is incorrect because pathologic prions are not in the α-helical configuration

29 Section 5 - Introduction to Fungi I. Fungi structure

Figure 5.1.25 - Fungi structure II. Types of fungi A. Two major types: 1. Filamentous (hyphae and mycelia) 2. Yeast

Figure 5.1.26 - Filamentous fungus

30

Photo Credit: (left) Bob Blaylock [CC BY-SA 3.0 (https://creativecommons.org/licenses/by-sa/3.0)]; (right) Microrao [Public domain]

Figure 5.1.27 - Mold (left) and yeast (right) III. Monomorphic and dimorphic A. Monomorphic 1. Fungi that only exist as one form (yeast or mold) B. Dimorphic 1. Fungi that exist in the form of both yeast and mold 2. Environmental conditions dictate morphological features 3. Mnemonic: “mold in the cold and yeast in the heat” C. Fungal spores 1. A dormant cell designed to survive harsh conditions 2. Structurally distinct from bacterial endospores 3. Less durable than bacterial endospores 4. Important for fungal asexual reproduction 5. Produced by almost all fungi

Photo Credit: CDC [Public domain]

Figure 5.1.28 - Spores of Aspergillus

31 REVIEW QUESTIONS 1. A 2-year-old boy has a diaper rash due to the fungal organism known as Candida albicans. Targeting of which of the following structures would be most helpful in treating this patient’s condition? A. B. C. D.

Mitochondria Ergosterol Cholesterol Protein synthesis

• Correct answer: B • This patient has a diaper rash caused by Candida albicans • The basis of the question is regarding which structure is unique to fungi • Ergosterol is present in the plasma membrane of fungi and is a unique molecule from human cells which makes it an important antifungal drug target (eg, nystatin) • A, C, and D are wrong because these are features of both fungi and human cells so targeting of these structures would most likely harm both types of cells and ultimately be unhelpful or even harmful

?

32 Section 6 - Viral Structures I. Viral structures A. Envelopes 1. Lipid bilayer that surrounds the capsid 2. Lack of envelope = naked virus B. Capsids 1. Protein structure that surrounds nucleic acids 2. Icosahedral 3. Helical

Figure 5.1.29 - Enveloped virus structures II. Viral entry A. Enveloped viruses 1. Adsorption: viral particles on lipid envelope attach to host cell 2. Penetration: lipid membranes fuse and viral capsid enters cell 3. Uncoating: capsid releases nucleic acids B. Naked viruses 1. Adsorption: viral particles on capsid attach to host cell 2. Penetration: cell brings in viral capsid through endocytosis

Figure 5.1.30 - Naked virus structures

3. Uncoating: capsid releases nucleic acids

33

Figure 5.1.31 - Adsorption, penetration, and uncoating of naked viruses

Figure 5.1.32 - Adsorption, penetration, and uncoating of enveloped viruses III. Viral disinfection using alcohol and chlorhexidine A. Disinfection: inactivation of most pathogens B. Sterilization: inactivation of all pathogens C. Alcohol and chlorhexidine destroy lipid membranes D. Enveloped viruses: lipid envelope destroyed → loss of viral particles → no adsorption → not infectious E. Naked viruses: viral particles intact on capsid → adsorption intact → infectious IV. Bacteriophages A. A virus that infects bacteria B. Responsible for the lytic cycle and the lysogenic cycle

Figure 5.1.33 - Bacteriophage structure

34 REVIEW QUESTIONS 1. A research study is evaluating the efficacy of disinfection using chlorhexidine. Every time a patient is discharged, pathogens present on counter surfaces are assessed and listed. Chlorhexidine is then used to wipe down the surface. Following disinfection, the researchers find that all of the bacteria are killed and most of the viruses are inactivated. However, some of the viruses remain viable after this procedure. The surviving viruses must possess which of the following? A. B. C. D.

A capsid protein for adsorption A bilayer made of phospholipids A double-stranded genome A single-stranded genome

• Correct answer: A • Chlorhexidine works primarily by disrupting lipid membranes (i.e. destruction of bacteria and enveloped viruses) • Naked viruses have proteins on their capsid that allow adsorption and entry into host cells • Choice B is incorrect because this describes enveloped viruses • Choices C and D are incorrect because naked viruses may be double-stranded or single-stranded, so this detail cannot be determined by the information provided

?

35 Section 7 - Viral Genetics I. Viruses can exchange genetic information with other viruses, leading to novel strains. Many genetic activities alter the genome of the virus, while other activities only lead to temporary phenotypic changes (eg, phenotypic mixing and complementation).

Genetic Activity

Genome Altered

Description

Recombination

Yes



Two chromosomes cross over at homologous regions

Reassortment (genetic shift)

Yes

• •

Segmented viruses exchange genetic material → genetic (antigenic) shift Example: pandemics due to influenza-related genetic shift

Random mutation (genetic drift)

Yes



Slow, gradual changes to the viral genome

• •

An accidental relationship Virus A and virus B infect the same cell → surface proteins from virus B are used to coat virus A Progeny: virus B surface proteins determine tropism (which cells they can infect) and surround virus A genome → host cells infected with progeny replicate only virus A genome → new progeny have original genotype and phenotype of virus A Example: HIV capsid within Rhabdovirus envelope

Phenotypic mixing

No

• •

Complementation

No

• • •

A dependent relationship Virus A and virus B infect the same cell → virus A provides products necessary for virus B to infect or replicate → virus B is now infectious but continually depends on virus A Example: HDV requires HBsAg for adsorption (HBV complements HDV)

Table 5.1.5 - Genetic adjustments and activities in viruses

Figure 5.1.34 - Genetic shift and genetic drift

36

Figure 5.1.35 - Phenotypic mixing

Figure 5.1.36 - HDV superinfection

37

?

REVIEW QUESTIONS 1. 1. Researchers isolate a virus (A) capable of invading and replicating within human pharyngeal tissue only. Pharyngeal tissue infected with virus A is examined and researchers confirm the presence of a second virus (B). Virus B is capable of infecting pharyngeal and intestinal tissues. Virus A and B have nonsegmented genomes. Viral progeny from the pharyngeal tissue have the nucleic acids of virus A with surface proteins from virus B. Which term best describes the hybrid progeny? A. B. C. D.

• Correct answer: B • Virus A genome coated with virus B surface proteins refers to phenotypic mixing • Virus B surface proteins will allow the hybrid virus A progeny to enter intestinal mucosal cells → tropism has altered

Genetic shift Altered tropism Dependence Genomic change

• Choice A is incorrect because genetic shift refers to reassortment which only occurs with segmented viruses • Choice C is incorrect because dependence refers to complementation, but virus A is capable of completing its life cycle without the surface proteins of virus B • Choice D is incorrect because phenotypic mixing is a temporary change, not a genomic change

38 Section 8 - Viral Genomes and Replication I. Viral genomes

III. Infectivity of purified viral genomes

A. RNA viruses are single-stranded (ssRNA) B. Reoviruses are double-stranded (dsRNA) C. DNA viruses are double-stranded (dsDNA) D. Parvoviruses are single-stranded (ssDNA) II. Positive and Negative Sense RNA A. Positive sense RNA 1. Resembles eukaryotic mRNA 2. Can be immediately translated upon entry to the host cytoplasm B. Negative sense RNA 1. Must first be transcribed to a positive sense strand 2. These viruses carry their own RNAdependent RNA polymerase to create the positive sense strand (eukaryotic cells lack RdRP)

Figure 5.1.37 - Positive-sense and negative-sense viruses

A. Purified nucleic acids = all but nucleic acids have been removed B. Infectious (if injected directly into host cells) following purification 1. Positive-stranded RNA viruses 2. DNA viruses (except poxviruses and HBV) C. Not infectious (even if injected into host cells) following purification 1. Negative-stranded RNA (lacks RdRP) 2. Poxviruses (requires its own DNAdependent RNA polymerase) 3. HBV (requires its own DNA polymerase)

39

Figure 5.1.38 - Poxvirus cytoplasm replication

Figure 5.1.39 - HBV structure IV. Translation and replication A. Translation → new proteins 1. Protein products include everything needed to make new virions (eg, capsids, viral particles for adsorption, polymerases for replication) B. Replication → copy viral genome 1. RNA viruses replicate in the cytoplasm 2. DNA viruses replicate in the nucleus (except for poxviruses)

Figure 5.1.40 - RNA virus replication

40 REVIEW QUESTIONS

?

1. Newly synthesized virions of a DNA virus obtain a lipid bilayer from the cell membrane of a eukaryotic host cell. This virus is not known to infect hepatic cells. Which of the following occurred at some point during the life cycle of this virus? A. DNA formation from an RNA-dependent polymerase B. RNA formation from an RNA-dependent polymerase C. Lipid membrane fusion with host cells D. Invagination of host cell membrane

Figure 5.1.41 - DNA virus replication V. Polymerases in viruses A. RNA-dependent RNA polymerase 1. Found only in negative sense RNA viruses 2. Negative strand is transcribed into a positive strand (mRNA) B. RNA-dependent DNA polymerase (reverse transcriptase) 1. Found only in retroviruses (HTLV and HIV) as well as HBV

VI. Envelope formation A. New virions will acquire their envelope from the plasma membrane of the host cell B. Herpesviruses acquire their envelope from the nuclear membrane

• Correct answer: C • The DNA virus in question is enveloped based on the fact that it obtained a lipid bilayer from the host cell • Enveloped viruses enter the host cell through fusion of their lipid membrane with the cytoplasmic membrane of the host cell • Choice A is incorrect because this describes reverse transcriptase, a polymerase used only by retroviruses (HIV and HTLV) and HBV • Retroviruses are RNA (the virus in question is DNA) • HBV infects hepatic cells (the virus in question does not infect hepatic cells) • Choice B is incorrect because this describes RNA-dependent RNA polymerase, which is carried only by negative-sense RNA viruses

• Choice D is incorrect because this describes endocytosis, which is how naked viruses enter host cells

BACTERIA

Figure 5.2.1 - Bacteria overview figure

41

42 Section 1 - Clostridium tetani

1. “Tethered to the gates of hell”: Clostridium tetani 2. Purple background: gram positive 3. Arched back: spasms of the spinal extensor muscles (arched back appearance) 4. Green gas: produces tetanospasmin (an exotoxin that causes muscle contractions) 5. Dice: diazepam can be used to reduce the muscle spasms 6. Missing jaw near lock of coffin: lockjaw 7. Snails with hard shells: spore-forming 8. Belly ring with dirty rope: unhygienic deliveries and umbilical cord care can be associated with infection 9. Mask: obligate anaerobe 10. Eerie grin: risus sardonicus (eerie grin)

11. Horns: tetanus toxin (tetanospasmin) travels to the anterior horn of the spinal cord 12. Monster snared to the ground: the toxin cleaves SNARE proteins 13. Gabe’s coffin: GABA is inhibited 14. Gliding Gabe: glycine is inhibited 15. Saw: renshaw cells 16. Rusty nails on robe: rusty nail puncture wounds may precede clinical symptoms 17. Clenched hand: clenched hands 18. Syringe in the hand: infection can be prevented with the tetanus vaccine 19. Y-shaped immunoglobulins: human tetanus immune globulin is used to treat an acute infection

43 REVIEW QUESTIONS

?

1. A 13-year-old unvaccinated boy presents to the emergency department due to involuntary muscle contractions. Physical examination reveals neck stiffness, clenched hands, and a decreased ability to open his jaw. The organism most likely responsible for this patient’s symptoms produces a toxin that cleaves what protein?

 

Photo Credit: Y tambe [GFDL (http://www.gnu.org/copyleft/fdl.html) or CC BY-SA 3.0 (https://creativecommons.org/licenses/by-sa/3.0)], via Wikimedia Commons

Figure 5.2.2 - Gram stain

Photo Credit: Content Provider(s): CDC/Don Stalons [Public domain], via Wikimedia Commons

Figure 5.2.3 - Gram-positive bacilli

Figure 5.2.4 - Tetanospasmin

• Correct answer: SNARE proteins • The patient is unvaccinated and is having involuntary muscle contractions • Decreased ability to open jaw → lockjaw • Clostridium tetani produces tetanospasmin which cleaves SNARE proteins

44 Section 2 - Clostridium botulinum

1. 2. 3. 4.

5. 6. 7. 8.

Underground robot fighting: Clostridium botulinum Purple in the background: gram positive Crime lord’s mask: obligate anaerobe Can-shaped robot: improperly canned food can cause infection in adults due to ingestion of preformed toxin Missing leg: descending paralysis Weaponized honey: ingestion of spores in honey can cause infant botulism Small, floppy robot: floppy baby syndrome M&M’s: spore-forming

9. Ensnared robot in honey: exotoxin cleaves SNARE proteins 10. Seed on head: toxin prevents release of acetylcholine 11. Open mouth: dysarthria 12. Crime lord’s sunglasses: blurred vision 13. Crime lord’s bowtie: botox injections can be used for therapeutic and cosmetic reasons 14. Fight ring with y-shaped spikes: treat with human botulinum immunoglobulin

45 REVIEW QUESTIONS

?

1. A 33-year-old female presents to the physician due to recurrent and chronic migraines. She has tried multiple medications but nothing seems to alleviate her symptoms. The physician recommends treatment with a medication known to cause focal paralysis of the injected region. This drug most likely prevents the release of what neurotransmitter?

Photo Credit: CDC/Dr. George lombard [Public domain]

Figure 5.2.5 - Gram stain of Clostridium botulinum

• Correct answer: acetylcholine • Medication being described is botox • Botox is used to treat chronic migraines and causes focal paralysis of the injected region • Botox is diluted Clostridium botulinum toxin so it must work by cleaving SNARE proteins associated with the release of acetylcholine into the synaptic cleft

46 Section 3 - Clostridium difficile

Difficult army training course: Clostridium difficile Purple background: gram-positive Flowers: normal intestinal flora Snails in the mud: spore-forming Mud tester wearing mask: obligate anaerobe Mud tester wearing mask and gloves: contact precautions including non-sterile gloves and a gown are important in preventing infection 7. Cleaning guy: prior antibiotic exposure (especially clindamycin) increases risk of infection 8. Poop pumper: patients taking PPIs are at an increased risk of infection 9. Drill sergeant “A”: toxin A (enterotoxin) 10. Bucket of poop: toxin A causes intestinal inflammation and fluid secretion 11. Drill sergeant “B”: toxin B (cytotoxin) 12. Drill sergeant “B” kills recruit: toxin B can induce cellular apoptosis 1. 2. 3. 4. 5. 6.

13. Skull on shoulders: toxin A and toxin B disrupt the integrity of the cytoskeleton within intestinal mucosal cells 14. Tire obstacles: toxin A and toxin B cause pseudomembranous colitis and diarrhea 15. Climbing chains: diagnosed using PCR (detects the genes that code for toxin B) 16. Mud tester collecting poop: diagnosed with a stool sample 17. Big army van: treat with oral vancomycin 18. Metro for recruit transportation: treat with metronidazole 19. Fit guy holding axe: treat with fidaxomicin 20. Transplanted flower: fecal microbiota transplant may be considered in recurrent cases

47 REVIEW QUESTIONS

?

1. A 73-year-old female is brought to the emergency department due to fevers, nausea, vomiting, and flank pain. Physical examination reveals costovertebral angle tenderness. After thorough evaluation she is treated with levofloxacin for pyelonephritis. Several days later her symptoms begin to resolve, but she develops watery diarrhea and abdominal pain. Nucleic acid amplification testing of the patient’s stool reveals the presence of the tcdB gene which encodes for a toxin produced by a gram-positive organism. Which of the following is most likely true regarding the toxin?

Photo Credit: CDC/Dr. Holdeman, Courtesy: Public Health Image Library

Figure 5.2.6 - Gram stain of Clostridium difficile

Photo Credit: User: Samir [CC BY 3.0 (https://creativecommons.org/licenses/ by/3.0)]

Figure 5.2.7 - Pseudomembranous colitis

A. B. C. D.

It activates adenylate cyclase It inactivates the 60S ribosomal subunit It disrupts the integrity of the cytoskeleton It cleaves SNARE proteins

• Correct answer: C • This patient has a C. diff infection • The patient was treated with antibiotics (levofloxacin) and then developed watery diarrhea • Antibiotic exposure (not just clindamycin) is a risk factor for C. diff infections • Nucleic acid amplification testing revealed the presence of a gene that encodes for a toxin produced by a grampositive organism • Toxin A and toxin B produced by C. diff both disrupt the integrity of the cytoskeleton • A is incorrect because this is describing the mechanism of other microorganisms (eg, Vibrio cholerae, enterotoxigenic E. coli, and Bacillus anthracis), but these are unlikely to suddenly cause symptoms in a patient recently treated with an antibiotic. • B is describing the mechanism of the toxin produced by Shigella and enterohemorrhagic E. coli • D is describing the mechanism of the toxin produced by Clostridium tetani and Clostridium botulinum but these are associated with muscle spasms or paralysis and not diarrhea.

48 Section 4 - Clostridium perfringens

Clown with perfume: Clostridium perfringens Purple background: gram positive Rice Krispie Treats: crepitus Man about to vomit: heat-labile enterotoxin causes food poisoning 5. Pot full of dirt: commonly found in soil 6. Alpha house sign: produces α toxin 7. Dum dum suckers stuck in pumpkin: the α toxin (lecithinase) is a phospholipase 1. 2. 3. 4.

8. Smashed pumpkin: degrades tissue and cell membranes (hemolysis) 9. Pumpkin seeds: spore-forming 10. Crow: causes myonecrosis 11. Gang of kids: causes gas gangrene 12. Kids wearing masks: obligate anaerobe 13. Murderer killing two people with two streaks of blood: “double zone” of hemolysis on blood agar 14. Janitor cleaning mess: treat with clindamycin

49 REVIEW QUESTIONS

?

1. A 27-year-old female presents to the emergency department due to a wound from a garden rake that penetrated her leg. Over the next several hours she develops intense pain and edema over the injured leg. Crepitus is present on physical examination. The organism most likely responsible for this patient’s condition produces a toxin with what mechanism of action? Photo Credit: Content Provider(s): CDC/Don Stalons [Public domain], via Wikimedia Commons

Figure 5.2.8 - Gram stain of Clostridium perfringens

Photo Credit: Engelbert Schröpfer, Stephan Rauthe and Thomas Meyer. [CC BY 2.0 (https://creativecommons.org/licenses/by/2.0)], via Wikimedia Commons

Figure 5.2.9 - Gas gangrene

• Correct answer: degrades the cell membrane • This patient sustained an injury from a dirty garden rake (wounds contaminated with dirt increase the risk of infection by Clostridium perfringens) • Crepitus is highly suggestive of Clostridium perfringens • Clostridium perfringens produces alpha toxin which is a phospholipase that degrades cell membranes

50 Section 5 - Bacillus anthracis

1. Giant ant hill: Bacillus anthracis 2. Purple tents: gram positive 3. Mounds extending away from ant hill: “Medusa head” 4. Wool coat and boots: increased risk in those who handle wool (woolsorter’s disease) 5. Girl holding inhaler: grows well under aerobic conditions 6. M&Ms on ground: spore-forming 7. Edamame: edema factor (exotoxin) 8. Tents: edema factor mimics adenylate cyclase (increases cAMP) 9. Backpack protecting girl from ants: protective antigen

10. Sign with skull: lethal factor 11. Ant “mates" glued together: polypeptide capsule (contains D-glutamate) 12. Coughing up red M&Ms: pulmonary anthrax causes pulmonary hemorrhage 13. M&Ms near mediastinum: pulmonary anthrax causes mediastinitis with widened mediastinum on CXR 14. Black ant biting hand: ulcer with black eschar seen in cutaneous anthrax 15. Flower staff: fluoroquinolones are an effective treatment

51 REVIEW QUESTIONS

?

1. A 37-year-old male is admitted to the hospital due to 3 days of fever, shortness of breath, and a recent episode of hemoptysis. He works as a shepherd during the summers, has no smoking history, and denies any recent travel. A sputum gram stain reveals gram-positive rods. Which of the following is most likely true regarding the microorganism?

Photo Credit: https://commons.wikimedia.org/wiki/File:Bacillus_anthracis_ Gram.jpg

Figure 5.2.10 - Gram stain of Bacillus anthracis

Photo Credit: Content Provider(s): CDC [Public domain], via Wikimedia Commons

Figure 5.2.11 - Cutaneous anthrax

A. The polysaccharide capsule is antiphagocytic B. One component of the exotoxin complex results in increased levels of cAMP C. The toxin inactivates elongation factor (EF2) D. It is an obligate anaerobe • Correct answer: B • This patient has pulmonary anthrax • Hemoptysis, shepherd work, and a gram stain showing gram-positive rods • Bacillus anthracis produces edema factor which mimics the effects of adenylate cyclase → ↑ cAMP • A is incorrect because while capsules are antiphagocytic, bacillus anthracis has a polypeptide capsule - not a polysaccharide capsule • C is incorrect as this is describing the Diphtheria toxin (EF-2 is associated with protein synthesis which is not directly affected by bacillus anthracis) • D is incorrect because bacillus anthracis is an aerobe - not an anaerobe

52 Section 6 - Bacillus cereus

Bowl of rice: Bacillus cereus Purple shirt: gram positive Rod-shaped walking stick: bacilli Hemp seeds: spore forming Cooking rice and hemp seeds over fire: spores can withstand being cooked in rice 6. Toxic green smoke: enterotoxin formation 7. “EMT” badge: emetic type 8. Eating rice: emetic type is associated with rice and pasta 1. 2. 3. 4. 5.

9. Rice cereal: emetic type produces cereulide toxin 10. Vomiting: emetic type causes vomiting 11. Five fingers: emetic type occurs 1-5 hours after ingestion of toxin 12. Guy sitting in puddle of water holding abdomen: diarrheal type results in watery diarrhea and gastrointestinal pain 13. “I voted” sticker: diarrheal type occurs 8-18 hours after ingestion of toxin

53 REVIEW QUESTIONS

?

1. An 8-year-old boy presents to the emergency department due to 6 hours of nausea, vomiting, and abdominal pain. He reports eating pasta at an Italian restaurant earlier today. He denies diarrhea, bloody stool, or recent travel. Physical examination reveals diffuse abdominal tenderness to palpation. Which of the following is most likely true regarding this patient’s condition?

Photo Credit: Y tambe [CC BY-SA 3.0 (https://creativecommons.org/licenses/ by-sa/3.0/)], via Wikimedia Commons

Figure 5.2.12 - Gram stain of Bacillus cereus

A. It is caused by an ingested pathogen that is actively producing a toxin B. It is due to a fecalith obstruction C. It is associated with Henoch-Schӧnlein purpura D. Microscopy will reveal noncaseating granulomas E. A preformed toxin is responsible • Correct answer: E • This patient has Bacillus cereus food poisoning • Acute onset nausea, vomiting, and abdominal pain after eating pasta • The spores can withstand being cooked and once the pasta is left at room temperature the spores germinate • A preformed toxin is produced that can rapidly cause symptoms when ingested • A is incorrect because organisms that actively produce toxins after ingestion usually don’t cause symptoms until 24 hours after the ingestion of the organism. • B is incorrect because this is describing appendicitis. • C is describing intussusception which is classically associated with red “currant jelly” stools • D is describing Crohn disease. This typically presents diarrhea as well as many other manifestations such as malabsorption problems, perianal disease, and dermatologic abnormalities

54 Section 7 - Listeria monocytogenes

Monocle: Listeria monocytogenes Purple colors: gram positive Rod-shaped bat: bacillus Narrow river and bay: narrow zone of β-hemolysis Cat: catalase positive Tumbleweed: tumbling motility in broth Whip cracking tail: forms “rocket tails” via actin polymerization 8. Man with gun inside of house: rocket tails facilitate intracellular movement and cell-to-cell spread across cell membranes 9. Asthma inhaler: aerobe 10. Building with no roof: facultative intracellular 11. People drinking milk and eating cheese and deli meats: acquired by ingestion of cold deli meats and unpasteurized dairy products 1. 2. 3. 4. 5. 6. 7.

12. Ice and cold water: grows well at cool temperatures (4-10 degrees C) 13. Candle: a TORCHES organism 14. Sick pregnant woman: septicemia in pregnant women 15. Vomit: self limited gastroenteritis in healthy individuals 16. Vomit on pregnant belly: amnionitis in pregnant women can lead to spontaneous abortion 17. Grandma: granulomatosis infantiseptica 18. Baby wearing hat: neonatal meningitis 19. Man on stretcher wearing hat: meningitis in immunocompromised 20. Old time amp: an effective treatment is ampicillin

55 REVIEW QUESTIONS

Photo Credit: CDC/Bobby Strong, Courtesy: Public Health Image Library

Figure 5.2.13 - Gram stain of gram-positive rods

Photo Credit: By Y tambe (Y tambe's file) [GFDL (https://www.gnu.org/ copylet/fdl.html), CC-BY-SA-3.0 (https://creativecommons.org/licenses/bysa/3.0/) or CC BY-SA 2.5 (https://creativecommons.org/licenses/by-sa/2.5/), via Wikimedia Commons

Figure 5.2.14 - Hemolysis on 5% sheep blood agar

Photo Credit: Philippinjl at French Wikipedia. [CC BY-SA 2.0 fr (https:// creativecommons.org/licenses/by-sa/2.0/fr/deed.en), via Wikimedia Commons

Figure 5.2.15 - Catalase test

?

1. A 23-year-old pregnant female is brought to the emergency department due to nausea, vomiting, and diarrhea. Several hours later she spontaneously delivers a preterm infant who has meningitis. Upon further investigation, the mother endorses regularly drinking unpasteurized milk throughout her pregnancy. Blood cultures from the mother and child reveal gram-positive rods that exhibit tumbling motility in broth. Which of the following would be most useful in eliminating the causal organism? A. B. C. D.

Cytotoxic T cell activity Antibody secretion Complement activation Ceftriaxone

• Correct answer: A • This mother and child have infections caused by Listeria monocytogenes • Listeria causes gastroenteritis in healthy individuals (eg, nausea, vomiting, and diarrhea) and meningitis in neonates • The mother has a history of drinking unpasteurized milk which is a risk factor for Listeria • Listeria is a gram-positive rod that exhibits tumbling motility in broth • Listeria is a facultative intracellular organism which means that it is protected from antibodies, complement, and other circulating immune factors • Elimination of the pathogen is reliant upon the activity of cytotoxic T cells which trigger cell lysis • B and C are both wrong because these are largely ineffective against an intracellular pathogen • D is incorrect because Listeria is resistant to third generation cephalosporins such as ceftriaxone

56 Section 8 - Corynebacterium diphtheriae

1. Haunted corn maze: Corynebacterium diphtheriae 2. Purple background: gram positive 3. Child with lice: must first be lysogenized by a bacteriophage before it produces a virulent exotoxin 4. Woman using asthma inhaler: aerobic 5. Woman using the telephone: tellurite agar 6. Alien: aniline dyes used to stain metachromatic granules 7. Mist by the corn maze: transmitted through aerosolized droplets 8. Tie around neck: pharyngitis 9. Bureau guy arresting business owner: the B subunit binds to host cell receptors and induces endocytosis of the toxin

10. Agency guy with extendable baton: the A subunit inhibits elongation factor 2 (EF-2) 11. Heat lamp: fever 12. Tires: pseudomembrane formation 13. Sausage-link rope: neurotoxic 14. FBI car: cardiotoxic (myocarditis, heart block, and arrhythmias) 15. Elk: Elek test 16. Laughing guy: Löffler’s medium 17. Syringe to pop tires: toxoid vaccine can prevent diphtheria 18. Y-shaped suspension tools: preformed antibody can be administered to help prevent systemic complications

57 REVIEW QUESTIONS

Photo Credit: Content Provider(s): [Public domain], via Wikimedia Commons

Figure 5.2.16 - Gram stain of Corynebacterium diphtheriae

1. A 12-year-old boy who recently moved to the United States from Nepal is brought to the emergency department due to fever, pharyngitis, and heart palpitations. His temperature is 38.5°C but other vital signs are normal. Physical examination is significant for a gray exudate on the posterior pharynx. A swab of the exudate is obtained. On microscopy there are gram-positive rods with granules that stain deeply with aniline dyes. He is admitted to the hospital. Which of the following should be done immediately to prevent additional complications? A. B. C. D. E.

Photo Credit: Nethan Hussain [CC BY-SA 3.0 (http://creative commons.org/ licenses/by-sa/3.0)], from Wikimedia Commons

Figure 5.2.17 - Cystine-tellurite agar

Photo Credit: User:Dileepunnikri [CC BY-SA 3.0 (https://creativecommons. org/licenses/by-sa/3.0)], from Wikimedia Commons

Figure 5.2.18 - Pseudomembranous pharyngitis

?

Careful monitoring of neurological status Intubation Serial electrocardiograms Preformed antitoxin administration Vaccine administration

• Correct answer: D • This patient has a Corynebacterium diphtheriae infection • Possible lack of immunization (eg, recently moved to the US from Nepal) • Gray exudate on the posterior pharynx • Gram-positive rods with granules that stain deeply with aniline dyes • The antitoxin contains antibodies that inactivate the diphtheria toxin (only useful before the toxin enters the cell so it must be administered immediately) • A is incorrect because administration of preformed antitoxin is more important (any delay may allow more of the toxin to enter the cells) • B is unnecessary at this point because the boy’s respiratory rate is normal and he doesn’t appear to be in any respiratory distress. • C is a good idea because diphtheria is cardiotoxic but just like choice A administration of the antitoxin is more important. • E should also be done but the immune response to vaccination is slow so the antitoxin should be given first in someone who is acutely ill

58 Section 9 - Viridans streptococci

1. Venomous villain: Viridans streptococci 2. Purple background: gram positive 3. Wide open mouth and tongue out: normal oropharyngeal flora 4. Mutant being: Streptococcus mutans 5. Mighty warrior: Streptococcus mitis 6. Teeth: Streptococcus mitis and Streptococcus mutans cause dental caries 7. Spider warrior: Streptococcus sanguinis 8. Alfalfa plants: α-hemolytic 9. Chin resisting web: optochin resistant

10. Spider webs adhering to buildings: produces dextrans which adhere to areas of endothelial trauma 11. Cars: subacute bacterial endocarditis 12. Teeth (dentistry): endocarditis after dental procedures in patients with pre-existing valvular lesions 13. Splinter in the foot: splinter hemorrhages 14. Green soda with ice cubes: bile insoluble 15. Pennies: treatment for endocarditis is penicillin G 16. Trident weapon: ceftriaxone

59 REVIEW QUESTIONS

Photo Credit: Y tambe [GFDL (http://www.gnu.org/copyleft/fdl.html), CC-BYSA-3.0 (http://creativecommons.org/licenses/by-sa/3.0/) or CC BY0SA 2.5 (https://creativecommons.org/licenses/by-sa/2.5)], via Wikimedia Commons

Figure 5.2.19 - Gram stain of Streptococci

Photo Credit: CDC: Dr. JJ Farmer (PHIL #3031), 1978. [Public domain], via Wikimedia Commons

Figure 5.2.20 - Microbial resistance and sensitivity to various compounds

Photo Credit: Splarka [Public domain], from Wikimedia Commons

Figure 5.2.21 - Splinter hemorrhages

?

1. A 29-year-old female presents to the physician due to intermittent fevers for the past 4 days. She states that she underwent a dental procedure 3 weeks ago, but did not receive prophylactic antibiotics. She has a history of a pre-existing valvular lesion which occurred several years ago. Examination of her fingers reveals dark lesions that run vertically underneath the nails beds. How will the causal organism most likely be affected by optochin? • Correct answer: unaffected by optochin • Intermittent fevers and dark lesions that run vertically underneath the nail beds (splinter hemorrhages) are suggestive of bacterial endocarditis • A history of a pre-existing valvular lesion and a recent dental procedure are suggestive of an infection caused by Viridans group streptococci • Viridans group streptococci are optochin resistant

60 Section 10 - Streptococcus pneumoniae

1. 2. 3. 4. 5. 6. 7. 8. 9.

New mower: Streptococcus pneumoniae Mower is purple: gram positive Alfalfa plant: α-hemolytic Hero that is sensitive about his chin: optochin sensitive Lances on mower: lancet-shaped diplococci Tridents: treat with ceftriaxone Scarecrow: treat with macrolides Quails: quellung reaction Sacks of apples: polysaccharide capsule

10. Syringe on the sack: the vaccine is directed against the polysaccharide capsule 11. Spilled green apple juice with melted ice cubes: bile soluble 12. Guy cleaning juice with mop: MOPS (meningitis, otitis media, pneumonia, and sinusitis) 13. Guy cutting down vegetation with a sickle: sepsis in sickle cell disease and asplenic patients 14. Scissors: IgA protease

61 REVIEW QUESTIONS

?

1. A 67-year-old female presents to the emergency department due to sudden onset nausea, vomiting, and confusion. Physical examination is significant for a fever of 38.6°C, nuchal rigidity, and altered mental status. Empiric treatment is started and a lumbar puncture is performed. CSF gram stain reveals gram-positive, lancetshaped diplococci. Administration of a vaccine directed against what bacterial structure may have been helpful in preventing infection?

Photo Credit: Content Provider(s): CDC [Public domain], via Wikimedia Commons

Figure 5.2.22 - Quellung reaction

Photo Credit: B. Welleschick [GFDL (http://www.gnu.org/copyleft/fdl.html), CC-BY-SA-3.0 (http://creativecommons.org/licenses/by-sa/3.0/) or CC BY-SA 2.5 (https:///creativecommons.org/licenses/by-sa-2.5)], from Wikimedia Commons

Figure 5.2.23 - Otitis media

• Correct answer: polysaccharide capsule • A fever, nuchal rigidity, and altered mental status → meningitis • Streptococcus pneumoniae is the most common cause of bacterial meningitis in adults • A CSF gram stain reveals gram-positive, lancet-shaped diplococci which confirms the diagnosis of Streptococcus pneumoniae • The vaccine targets the polysaccharide capsule

62 Section 11 - Streptococcus pyogenes

Pie: Streptococcus pyogenes Purple floor: gram positive House on the bay: β-hemolytic “Put your raincoat”: PYR positive Covered pie: hyaluronic acid capsule Burned hand: skin infections (cellulitis) Honey on the pie: impetigo Red mitten: erysipelas Superhero cape: pyrogenic exotoxin (a superantigen) causes toxic shock-like syndrome 10. Shocked baby: toxic shock-like syndrome causes shock 11. Sweating baby: toxic shock-like syndrome causes fever 12. Red rash on baby’s belly: toxic shock-like syndrome causes a rash 13. Woman wearing scarf: pyrogenic exotoxin causes scarlet fever 14. Crumbs that resemble sand: scarlet fever causes a sandpaper-like rash 15. Scarf around neck: scarlet fever causes pharyngitis 16. Munching on a strawberry: strawberry tongue 17. Burnt pie: necrotizing fasciitis 18. O-shaped donuts that are filled with red jelly: streptolysin O is a toxin that degrades cell membranes and lyses RBCs 1. 2. 3. 4. 5. 6. 7. 8. 9.

19. “S” is for strawberry: streptolysin S 20. Guy with a fever and towel on his head: rheumatic fever 21. “Welcome to the JONES’ sign”: JONES 22. Donut jelly flying to guy with a fever: antistreptolysin O (ASO) titers can be used to diagnose rheumatic fever 23. Maid: M protein is a virulence factor 24. Maid quieting down noisy kids: M protein inhibits phagocytosis 25. Maid caring for guy with fever: M protein can give rise to rheumatic fever 26. Toy cars by maid: M protein is associated with carditis 27. Tangled up cord in shape of glomerulus: poststreptococcal glomerulonephritis 28. Broken bass guitar: bacitracin sensitive 29. Stressed out guy pulling on tie wrapped around neck: pharyngitis 30. Pennies on the poker table: penicillin G used to treat pharyngitis 31. Sapphires on the poker table: cephalexin used to treat skin infections (impetigo)

63

Photo Credit: RafaelLopez at the English language Wikipedia[CC BY-SA 3.0 (http://creativecommons.org/licenses/by-sa/3.0/)]

Figure 5.2.24 - Cellulitis

Photo Credit: Evanherk at Dutch Wikipedia [CC BY-SA 3.0 (http:// creativecommons.org/licenses/by-sa/3.0/)]

Figure 5.2.25 - Impetigo

64

Photo Credit: CDC/Dr. Thomas F. Sellers/Emory University [Public domain]

Figure 5.2.26 - Erysipelas

Photo Credit: https://en.wikipedia.org/wiki/File:Scarlet_Fever.jpg#file

Figure 5.2.27 - Sandpaper-like rash

Photo Credit: Afag Azizova [CC BY-SA 3.0 (http://creativecommons.org/ licenses/by-sa/3.0/)]

Figure 5.2.28 - Strawberry tongue

Photo Credit: https://en.wikipedia.org/wiki/File:Pos_strep.JPG#file

Figure 5.2.29 - GAS pharyngitis

65 REVIEW QUESTIONS 1. A 12-year-old boy comes to the office due to 2 days of a sore throat. A rapid antigen detection test is positive for group A Streptococcus (GAS). The physician immediately prescribes penicillin due to concerns about a complication that would occur if the infection was left untreated. The complication occurs due to a virulence factor produced by the organism which subsequently triggers an immune response. What is the most likely function of the virulence factor described? • Correct answer: resists phagocytosis • The patient has group A strep (GAS) and is prescribed penicillin due to concerns of developing rheumatic fever • Rheumatic fever is caused by M-protein which is a virulence factor that triggers an immune response and causes the host to produce antibodies that target bacterial antigens • The bacterial antigens may be similar to antigens present in the heart which means that host antibodies may accidently target the heart • Additionally, M-protein resists phagocytosis

?

66 Section 12 - Streptococcus agalactiae

1. 2. 3. 4. 5. 6. 7. 8. 9.

Glacier: Streptococcus agalactiae Bay: β-hemolytic Purple tents: gram positive Camping: CAMP factor Sleep sack: polysaccharide capsule Bass guitar with chains: bacitracin resistant Hippies: positive hippurate test Pregnant lady coughing: pneumonia in neonates Guy sipping hot chocolate near pregnant lady: sepsis in neonates

10. Hat and headband near pregnant lady: meningitis in neonates 11. Pregnant lady: screen pregnant women at 35-37 weeks gestation 12. Pennies: pregnant women colonized with GBS receive intrapartum penicillin prophylaxis 13. Amp: pregnant women may also be treated with ampicillin

67 REVIEW QUESTIONS 1. A 27-year-old female presents to the physician for a follow-up visit at 37 weeks gestation. The physician recommends vaginal and rectal cultures to screen for colonization of a potentially harmful microorganism transmitted to the neonate during childbirth. The organism of interest most likely demonstrates what type of hemolysis?

Photo Credit: By Clueiridium [CC0], from Wikimedia Commons

Figure 5.2.30 - CAMP test

• Correct answer: β-hemolysis • GBS may colonize the vagina and rectum so vaginal and rectal cultures are typically obtained at 37 weeks gestation • GBS can be transmitted to the neonate during childbirth • GBS exhibits β-hemolysis on sheep blood agar

?

68 Section 13 - Streptococcus gallolyticus

1. 2. 3. 4. 5.

Galileo: Streptococcus gallolyticus Purple colors: gram positive Flowers near brown river: normal colonic flora Green gamma ray: γ-hemolytic Green reflection in river: grows in bile

6. Bloody wound: bacteremia 7. Cart: endocarditis 8. Cancer hope ribbon near muddy river: associated with colon cancer

69

?

REVIEW QUESTIONS 1. A 33-year-old male with a history of IV drug use comes to the office due to recent changes in his fingernail. Examination of the fingernail is shown below. Physical examination is also significant for a holosystolic high-pitched murmur that is best heard near the left sternal border. Blood cultures are positive for an organism that grows well in bile but fails to grow in the presence of 6.5% NaCl. Which of the following is true regarding the organism responsible for this patient’s condition?

Photo Credit: Splarka [Public domain], from Wikimedia Commons

A. B. C. D.

It is catalase positive It is a gram-positive rod It is associated with colon cancer It is associated with abdominal wound infections E. It exhibits α-hemolysis on blood agar • Correct answer: C • The patient has endocarditis • IV drug use • Splinter hemorrhages (shown in the image) • Cardiac murmur • Blood cultures reveal an organism that grows well in bile but not in the presence of 6.5% NaCl → organism must be Streptococcus gallolyticus (Enterococci grow in bile and 6.5% NaCl)

• Streptococcus gallolyticus can cause endocarditis and is associated with colon cancer • A is false - Streptococcus gallolyticus is catalase-negative (Staphylococcus aureus  is catalase-positive and can cause endocarditis but does not grow in the presence of bile) • B is false - Streptococcus gallolyticus is a gram-positive coccus • D is false - it’s not typically associated with abdominal wound infections (this is true of Enterococcus) • E is false - Streptococcus gallolyticus exhibits γ-hemolysis on blood agar

70 Section 14 - Enterococci

1. 2. 3. 4. 5. 6. 7. 8. 9.

“Enter” sign: Enterococcus Purple colors: gram positive Flowers near sewer: normal intestinal flora “Put Your Raincoat”: PYR positive Salt shaker: grows in 6.5% NaCl Green gamma rays: γ-hemolytic Green soda near growing flowers: grows in bile Punched in the belly: abdominal wound infections Spilled fluid on crotch: urinary tract infections

10. Car: endocarditis 11. Charging attendees a fee: Enterococcus faecium and Enterococcus faecalis 12. Banana trees: biliary tree infections 13. Amp: treat with ampicillin 14. Line tape: treat with linezolid 15. Resistant bouncer guys near van: vancomycin resistant

71 REVIEW QUESTIONS

Photo Credit: Content Provider(s): CDC/Dr. Richard Facklam [Public domain]

Figure 5.2.31 - Gram stain of Enterococci

?

1. A 72-year-old male is admitted to the hospital for community acquired pneumonia. A urinary catheter is placed upon admission because the patient is unable to ambulate to the bathroom. After two days of appropriate treatment his symptoms begin to resolve. However, on the third day of admission he begins to experience dysuria, frequency, and urgency. A urine culture grows gram-positive cocci in pairs and chains. Antibiotic sensitivity testing reveals that the organism is resistant to vancomycin. The causal organism will most likely demonstrate which of the following? A. Beta hemolysis on blood agar B. PYR positivity C. Absence of growth in hypertonic (6.5%) saline D. Water and oxygen production in the presence of hydrogen peroxide • Correct answer: B • Reasons why Enterococcus is the most likely organism: • UTI (dysuria, frequency, and urgency following placement of a catheter) • Morphology of the organism (grampositive cocci in pairs and chains) • Ability to resist vancomycin • Enterococcus is PYR positive • A is wrong because Enterococcus is γ-hemolytic • C is wrong because Enterococcus grows well in the presence of 6.5% NaCl • D is describing the catalase test and Enterococcus is catalase-negative

72 Section 15 - Staphylococcus aureus

Merlin’s staff: Staphylococcus aureus Purple background: gram positive Merlin’s cat: catalase positive Bay: β-hemolytic Tall man: ferments mannitol “A” on shield: protein A prevents opsonization and phagocytosis 7. Fire coming out of dragon’s nose: colonizes the nares 8. Coagulated blood: coagulase positive 9. Fire scalding man’s skin: scalded skin syndrome 10. Man vomiting: rapid-onset food poisoning 11. Guy getting shocked: toxic shock syndrome 12. Guy getting shocked is wearing a superhero cape: TSST-1 is a superantigen (causes binding of MHC class II and T cell receptors → ↑ cytokine release → shock) 1. 2. 3. 4. 5. 6.

13. Tissue packed in nose: classically associated with prolonged use of a tampon or nasal packing 14. Guy vomiting: vomiting 15. Guy is getting shocked: shock 16. Rash with skin falling off: desquamating rash 17. Dragon scales: osteomyelitis 18. Red knees: septic arthritis 19. Guy near mosquitos is coughing due to smoke: postviral bacterial pneumonia 20. Burns on arm: skin infections and abscesses 21. Chest of myrrh: MRSA 22. Bricks (i.e. altered wall) on ground causing myrrh chest to fall off of cart: MRSA have altered penicillin binding proteins 23. Caravan: vancomycin is effective against MRSA 24. Sapphires: cephalexin for non-MRSA infections 25. Vials of drugs on cart: endocarditis in IV drug users

73 REVIEW QUESTIONS

Photo Credit: By Y Tambe (Y Tambe) [GFDL (http://www.gnu.org/copyleft/fdl. html) or CC-BY-SA-3.0 (http://creativecommons.org/licenses/by-sa/3.0/)], via Wikimedia Commons

Figure 5.2.32 - Gram stain of Staphylococcus

?

1. A 27-year-old female presents to the emergency department due to a 5-hour history of fever, rash, and altered mental status. Her blood pressure is 76/58 and she is tachycardic. She is examined thoroughly and a used tampon is removed from the vagina. A gram stain of a nasal swab specimen reveals gram-positive cocci in clusters. The organism most likely responsible for this patient’s presentation produced the systemic syndrome through which of the following mechanisms? A. Overactivation of cAMP B. Lecithinase-mediated degradation of tissue C. Toxin B-mediated disruption of the cytoskeleton D. Excessive release of cytokines

Photo Credit: Navaho [CC BY-SA 4.0 (https://creativecommons.org/licenses/ by-sa/4.0)]

Figure 5.2.33 - Mannitol fermentation test

Photo Credit: Philippinji [GFDL (http://www.gnu.org/copyleft/fdl.html) or CCBY-SA-3.0 (http://creativecommons.org/licenses/by-sa/3.0/)], via Wikimedia Commons

Figure 5.2.34 - Coagulase test tube

• Correct answer: D • Reasons why Staphylococcus aureus is the most likely organism: • The patient has toxic shock syndrome (fever, rash, altered mental status, hypotension, and the presence of a used tampon in the vagina) • A nasal swab showed gram-positive cocci in clusters • Toxic shock syndrome is caused by toxic shock syndrome toxin 1 (TSST-1) • This is a superantigen which causes binding of MHC class II and T-cell receptors resulting in overactivation of T-cells → excessive release of cytokines • A is true of several toxins but Staphylococcus aureus does not affect cAMP • B is describing the alpha toxin of Clostridium perfringens • C is describing the toxin of Clostridium difficile

74 Section 16 - Staphylococcus epidermidis

Dermatology clinic: Staphylococcus epidermidis Purple background: gram positive Flowers: normal skin flora Bison vulnerably sitting on operating table: novobiocin sensitive 5. Spattered bison blood: frequently contaminates blood cultures 1. 2. 3. 4.

6. Cat: catalase positive 7. Urine: urease positive 8. Biofilm on the pipe: produces an adherent biofilm 9. Foreign body stuck in bison: infects foreign bodies 10. Tools with tubes: infects IV catheters 11. Car: can cause prosthetic valve endocarditis 12. Van: vancomycin is an effective treatment

75

Photo Credit: https://www.flickr.com/photos/36349503@N08/3368611155

Figure 5.2.35 - Urease test

?

REVIEW QUESTIONS 1. A 39-year-old male is admitted to the hospital due to a fever of 38.7 C and a cardiac murmur. He recently had his aortic valve replaced due to degeneration of a congenital bicuspid aortic valve. Blood cultures are obtained and grow coagulase-negative staphylococci. Which of the following is most likely true regarding this organism? A. B. C. D.

It produces an adherent biofilm It releases an exotoxin into the bloodstream It is urease negative It is novobiocin resistant

• Correct answer: A • This patient has a Staphylococcus epidermidis infection causing prosthetic valve endocarditis • Coagulase-negative staphylococcus on gram stain • The patient has a fever, heart murmur, recent aortic valve replacement, and a positive blood culture → prosthetic valve endocarditis caused by Staphylococcus epidermidis

• Staphylococcus epidermidis produces an adherent biofilm that is made up of an extracellular polysaccharide matrix that acts as a barrier to antibiotics and the immune system • B is false because Staphylococcus epidermidis doesn’t produce an exotoxin • C is D are false because Staphylococcus epidermidis is urease positive and novobiocin sensitive (Staphylococcus saprophyticus is novobiocin resistant)

76 Section 17 - Staphylococcus saprophyticus

1. Sap: Staphylococcus saprophyticus 2. Purple shirt: gram positive 3. Young attractive girl near urine: UTI in sexually active women 4. Bison and chain: novobiocin resistant 5. Cat: catalase positive 6. Urine: urease positive

7. Flowers covered in urine: normal flora of the female genital tract 8. Fury toes: nitrofurantoin is a first line agent for uncomplicated UTIs 9. Meth crystals in sap: TMP-SMX is an effective treatment

77 REVIEW QUESTIONS 1. A 19-year-old sexually active female presents to the office with dysuria, urgency, and frequency. Urine cultures reveal gram-positive cocci. What two lab tests would be most helpful in distinguishing the causal organism from other members of the genus? A. The catalase test and the urease test B. The catalase test and the type of hemolysis observed on blood agar C. The type of hemolysis observed on blood agar and the coagulase test D. The coagulase test and the catalase test E. The coagulase test and the novobiocin test • Correct answer: E • This patient has a UTI caused by Staphylococcus saprophyticus • Dysuria, urgency, and frequency → UTI • Young, sexually active, and urine cultures have revealed gram-positive cocci → Staphylococcus saprophyticus • Staphylococcus saprophyticus, Staphylococcus aureus, and Staphylococcus epidermidis belong to the same genus and can be distinguished from each other using the coagulase test and the novobiocin test • See picture below (A, B, C, & D)

?

78 Section 18 - Nocardia

1. Notecards: Nocardia 2. Purple colors: gram positive 3. Tangled up cords: branching filamentous morphology 4. Dirt: endemic in soil 5. Inhaler: obligate aerobe 6. Microphone: mycolic acid 7. Acid melting door rapidly: weakly acid-fast 8. Car bomb: carbol fuchsin

9. Urine: urease positive 10. Cat: catalase positive 11. Stretcher: predominantly occurs in immunocompromised individuals 12. Burned skin: cutaneous involvement 13. Cough: pneumonia 14. Burning red car parts on top of the building: brain abscesses 15. Meth crystals: treat with TMP-SMX

79 REVIEW QUESTIONS

Photo Credit: CDC/Dr. Lucille K. George, Courtesy: Public Health Image Library

Figure 5.2.36 - Gram stain of Nocardia

Photo Credit: CDC/Dr. George P. Kubica [Public domain], via Wikimedia Commons

Figure 5.2.37 - Acid-fast stain of Mycobacterium Tuberculosis (similar to Nocardia)

?

1. A 33-year-old homeless male is brought to the emergency department due to a severe headache and altered mental status. He has a history of HIV and has not been compliant with his medications. The patient also has had a fever, night sweats, and a productive cough for the past several weeks. Physical examination reveals bilateral papilledema and crackles heard throughout all lung fields. An MRI of the brain reveals an abscess. Which of the following is true regarding the most likely causal organism? A. B. C. D.

It is a gram positive coccus It is a gram negative rod It is catalase positive It is an obligate anaerobe

• Correct answer: C • This patient has a brain abscess caused by Nocardia • Immunocompromised • Noncompliant with medications • Fever • Night sweats • Cough • Brain abscess • Nocardia is catalase positive • A is false because while Staphylococcus aureus is a more common cause of brain abscess, the patient’s other symptoms (eg, history of HIV, cough) make Nocardia more likely • B is false because Nocardia is a filamentous, gram-positive rod • D is false because Nocardia is an obligate aerobe

80 Section 19 - Actinomyces israelii

1. Actor: Actinomyces israelii 2. Purple colors: gram positive 3. Tangled up cords: branching filamentous morphology 4. Mask: anaerobic 5. Yellow sand in air: yellow “sulfur” granules 6. Flowers: part of the normal oral, reproductive, and GI flora

7. Skull with open mouth: colonizes the mouth, is found in dental caries, and can cause cervicofacial disease 8. Movie director guy aspirating coffee: pulmonary actinomyces may develop following aspiration 9. Devices: intrauterine device infection can result in PID 10. Pennies: treat with penicillin

81 REVIEW QUESTIONS

?

1. A 42-year-old female comes to the office due to a chronic, slowly progressive, indurated mass on the left side of her face. Physical examination reveals a nontender mass near the left aspect of the mandible. An aspiration of the mass is sent for further analysis and a gram stain is shown below.

Photo Credit: Yale Rosen from USA [CC BY-SA 2.0 https://creativecommons. org/licenses/by-sa/2.0)]

Figure 5.2.38 - Gram stain of Actinomyces

Photo Credit: Yale Rosen from USA [CC BY-SA 2.0 https:// creativecommons.org/licenses/by-sa/2.0)]

A. It should be urgently treated with surgery B. It is caused by an aerobic organism C. The causal organism is part of the normal oral flora D. It stains weakly acid fast Photo Credit: Yale Rosen from USA [CC BY-SA 2.0 https://creativecommons. org/licenses/by-sa/2.0)]

Figure 5.2.39 - Sulfur granule

• Correct answer: C • This patient has a cervicofacial abscess caused by Actinomyces israelii • Cervicofacial abscess affecting the mandible is likely Actinomyces israelii which comes from normal oral flora • The gram stain shows a gram-positive organism with a central sulfur granule which is characteristic of Actinomyces israelii • A is false because the description of her infection wouldn’t require urgent surgery • B is false because Actinomyces israelii is an anaerobic species • D is false because Actinomyces israelii doesn’t stain weakly acid-fast (Nocardia does)

82 Section 20 - Neisseria overview

1. Knife: Neisseria 2. Pink colors: gram negative 3. Two pieces of candy being dipped into the chocolate bowl: diplococci 4. Prominent tall hat: Neisseria meningitidis 5. Gondola: Neisseria gonorrhoeae 6. Chocolate candy in chocolate factory: grows on chocolate agar 7. Lips: contains lipooligosaccharides (strong endotoxins) 8. Pillars and packages of various shapes and sizes: displays antigenic variation of pili 9. Scissors: IgA protease

10. Glue: ferments glucose 11. Oompa loompas: opa-proteins 12. “OPEN 9-5” sign: C5-C9 complement deficiency increases susceptibility to recurrent neisseria infections 13. Blue necklace: oxidase positive 14. Thayer-Martin chocolate food van: grows in ThayerMartin agar 15. Van: vancomycin is present in Thayer-Martin agar 16. Nice Satan: nystatin is present in Thayer-Martin agar 17. Mixer: polymyxin is present in Thayer-Martin agar 18. Meth candy bar with three segments: trimethoprim  is present in Thayer-Martin agar

Photo Credit: Content Provider(s): CDC/Dr. Norman Jacobs [Public domain], via Wikimedia Commons

Photo Credit: CDC/Megan Mathias and J. Todd Parker [Public domain], via Wikimedia Commons

Figure 5.2.40 - Gram stain of Neisseria

Figure 5.2.41 - Chocolate agar

83

Photo Credit: Jeff Dahl [GFDL (http://www.gnu.org/copyleft/fdl.html) or CC BY-SA 4.0 (https://creative commons.org/licenses/by-sa/4.0)], from Wikimedia Commons

Figure 5.2.42 - Gram negative bacterial cell wall

Photo Credit: http://commons.wikimedia.org/wikiFile: Cytoxydase. jpg#filehistory

Figure 5.2.43 - Oxidase test

Photo Credit: https://commons.wikimedia.org/wiki/File:Neisseria_ gonorrhoeae_01.png

Figure 5.2.44 - Chocolate agar and Thayer-Martin agar

REVIEW QUESTIONS

?

1. A 26-year-old male presents to the office due to urethral discharge. One of his partners was recently diagnosed with a sexually transmitted infection. A swab of the discharge is sent for further laboratory analysis. The pathologist cultures the sample on a selective medium in an attempt to isolate the causal organism. Which of the following compounds is likely present in the medium? A. B. C. D. E.

Cephalexin Penicillin Azithromycin Vancomycin Caspofungin

• Correct answer: D • The patient presentation is suggestive of a Neisseria gonorrhoeae infection • A selective medium (which isolates a particular organism) for Neisseria gonorrhoeae is the Thayer-Martin agar • Thayer-Martin agar contains vancomycin, nystatin, polymyxin, and trimethoprim • A, B, C, and E are false because they are not present in Thayer-Martin agar

84 Section 21 - Neisseria meningitidis

1. Knife and top hat: Neisseria meningitidis 2. Pink colors: gram negative 3. Picture of Neisseria overview image: don’t forget the overview information 4. Mist: transmitted through respiratory secretions 5. Sack: polysaccharide capsule 6. Syringe next to ammo sacks: the vaccine contains antigens from the polysaccharide capsule 7. Horse with prominent nostrils: colonizes the nares 8. Malt beer: maltose fermenter 9. Sipping: sepsis 10. Red spots on horse: petechial rash 11. Hats: meningitis 12. Black foot and toes: gangrene of the toes 13. Putting fire out with a bucket of water: WaterhouseFriderichsen syndrome

14. Chimney shaped like a triangle: WaterhouseFriderichsen syndrome causes adrenal insufficiency 15. Lamp next to house: Waterhouse-Friderichsen syndrome is associated with a fever 16. Coagulated jello: Waterhouse-Friderichsen syndrome is associated with DIC 17. Stray wire and shock symbol: WaterhouseFriderichsen syndrome is associated with shock 18. Union soldier stabbing guy in the spleen: asplenic patients are at an increased risk of developing an infection 19. Pennies: treat with penicillin G 20. Trident: treat with ceftriaxone 21. Rifles with bayonets for close combat: close contacts are prophylactically treated with rifampin

85 REVIEW QUESTIONS 1. A 45-year-old female is brought to the emergency department due to fever, a petechial rash, and altered mental status. Her past medical history is significant for an ATV accident in which she sustained trauma to the abdomen that required surgery due to internal bleeding. Her blood pressure is 72/41 and heart rate is 129/min. Gram stain from a lumbar puncture reveals gram-negative diplococci. Impairment of which of the following most likely contributed to this patient’s condition? A. B. C. D.

Reactive oxygen species Immunoglobulin production Complement production Microtubule formation

• Correct answer: B • The patient has a history of abdominal trauma with significant internal bleeding that required surgery - this suggests she had a splenectomy following splenic rupture • Her current presentation is suggestive of Waterhouse-Friderichsen syndrome from an infection with Neisseria meningitidis • The absence of a spleen makes infection from encapsulated organisms more likely (Neisseria meningitis has a polysaccharide capsule) • The splenic B-lymphocytes produce a large percentage of the body’s immunoglobulins which help with opsonization (IgG) and removal of encapsulated organisms • A is false because an inability to form reactive oxygen species makes patients more susceptible to catalase positive organisms - as in NADPH deficiency • C is false because the liver is where complement proteins are produced • D is false because microtubule dysfunction is associated with drug effects

?

86 Section 22 - Neisseria gonorrhoeae

1. Gondola: Neisseria gonorrhoeae 2. Pink colors: gram negative 3. Neisseria overview image: don’t forget the overview information 4. Unique pillars: antigenic variation of pili (evades immune system and has prevented vaccine development) 5. Scarlet letter: sexually transmitted infection 6. Peeing off ledge: dysuria and penile discharge in males 7. Pee hitting eye: neonatal conjunctivitis 8. Fist: Fitz-Hugh-Curtis syndrome

9. Pregnant woman outside of the gondola: increased risk of ectopic pregnancy 10. Uterus-looking chandelier: pelvic inflammatory disease 11. Tendons about to burst: tenosynovitis 12. Glass breaking around knees: septic arthritis 13. Clam: chlamydial coinfection 14. Gnat bugs: diagnosis made with nucleic acid amplification test (NAAT) 15. Trident: treated with ceftriaxone 16. Scarecrow: treated with macrolides (for chlamydial coinfection)

87 REVIEW QUESTIONS 1. A 24-year-old male presents to the clinic due to dysuria. He states that his symptoms began two days ago. He is sexually active with multiple partners. Physical examination reveals a white discharge expressed from the penis. Gram stain of the discharge reveals gram-negative diplococci seen in the cytoplasm of neutrophils. Which of the following is the recommended treatment regimen for the causal organism? A. B. C. D. E.

Doxycycline and cephalexin Ceftriaxone and tigecycline Trimethoprim-sulfamethoxazole Ceftriaxone and azithromycin Penicillin and clavulanic acid

• Correct answer: D • The patient has urethritis caused by Neisseria gonorrhoeae • Dysuria • Multiple partners • White discharge expressed from the penis • Gram stain showing gram-negative diplococci in the cytoplasm of neutrophils • Effective treatment involves using both ceftriaxone and azithromycin  (ceftriaxone is used to cover gonococcal infection and azithromycin is used to cover a possible chlamydia coinfection) • A is incorrect because doxycycline can be used to treat the possible Chlamydia coinfection but cephalexin is a 1st generation cephalosporin and not commonly used to treat STIs • B is false because tigecycline is used as a broad-spectrum antibiotic for Staphylococcus aureus skin and soft tissue infections as well as MRSA and VRE • C and E are false because both trimethoprim-sulfamethoxazole and penicillin with clavulanic acid are not used to treat Neisseria gonorrhoeae or Chlamydia

?

88 Section 23 - Moraxella catarrhalis

1. Cactus: Moraxella catarrhalis 2. Red sunset: gram negative 3. Flowers on top of cacti: normal flora of the respiratory tract 4. Dusty mist: transmitted through respiratory droplets 5. Inhaler: aerobe

6. Colorado Police Department: COPD exacerbations 7. Earmuffs on child: otitis media in children 8. Signs: sinusitis 9. Ammo: amoxicillin is an effective treatment 10. Cleaver: an effective treatment is amoxicillin and clavulanic acid

89 REVIEW QUESTIONS

?

1. A 16-month-old boy is brought to the physician due to increased irritability, fever, and poor feeding. Physical examination reveals an erythematous bulging tympanic membrane. Which of the following is the most likely causal organism?

Photo Credit: CDC/Dr. W.A. Clark, Courtesy: Public Health Image Library

Figure 5.2.45 - Gram stain of Moraxella

Photo Credit: Welleschik [GFDL (http://www.gnu.org/copyleft/fdl.html), CC-BY-SA-3.0 (http://creativecommons.org/licenses/by-sa/3.0) or CC BY-SA 2.5 (https://creativecommons/org/licenses/by-sa/2.5)], from Wikimedia Commons

Figure 5.2.46 - Otoscopic view of otitis media

A. B. C. D.

Moraxella catarrhalis Streptococcus pneumoniae Haemophilus influenzae Staphylococcus aureus

• Correct answer: B • Streptococcus pneumoniae is the most common cause of otitis media in children (approximately 50% of cases) • Moraxella catarrhalis accounts for less than 10% of cases (A) • Haemophilus influenzae accounts for about 45% of cases (C) • Staphylococcus aureus is a rare cause of otitis media in children (D)

90 Section 24 - Haemophilus influenzae

Office: Haemophilus influenzae Pink walls: gram negative Scissors: IgA protease Jar of chocolate: grows on chocolate agar Staff: can be grown near Staphylococcus aureus on blood agar 6. Star fish with five points: factor V is necessary for the growth of Haemophilus influenzae 7. “10 RULES OF THE OFFICE” sign: factor X is necessary for the growth of Haemophilus influenzae 8. “PLEASE RESPECT PROPERTY” sign and stapler surrounded by jello: encapsulated strains have a polyribosylribitol phosphate (PRP) polysaccharide capsule 9. Tape dividing the room in half: there are typeable and nontypeable strains of Haemophilus influenzae 10. Syringe: there is a vaccine for Haemophilus influenzae serotype b (Hib) 11. Dwayne opening mouth and yelling “Michael”: Haemophilus influenzae serotype b (Hib) causes epiglottitis 12. Computer screen that has the “thumb sign”: “thumb sign” seen on lateral neck x-ray 1. 2. 3. 4. 5.

13. Michael with red knees: Haemophilus influenzae serotype b (Hib) causes septic arthritis 14. Hood: Haemophilus influenzae serotype b (Hib) causes meningitis 15. Sickle: asplenia increases the risk of Haemophilus influenzae serotype b (Hib)  infections 16. Trident: ceftriaxone can be used to treat Haemophilus influenzae serotype b (Hib) infections 17. Rifle on shirt: rifampin prophylaxis for close contacts 18. Broken keyboard: nontypeable strains (not encapsulated) 19. Smoke: aerosolized transmission 20. Signs: nontypeable strains of Haemophilus influenzae cause sinusitis 21. Ear muffs: nontypeable strains of Haemophilus influenzae cause acute otitis media 22. Red eyes: nontypeable strains of Haemophilus influenzae cause conjunctivitis 23. Ammo and a cleaver: nontypeable strains of Haemophilus influenzae are treated with amoxicillin and clavulanate

91

Photo Credit: Dimitrios Malamos [CC BY 4.0 (https://creativecommons.org/ licenses/by/4.0)] Photo Credit: Bobjgalindo [GFDL (http://www.gnu.org/copyleft/fdl.html) or C BY-SA 4.0 (https://creativecommons.org/licenses/by-sa/4.0)], from Wikimedia Commons

Figure 5.2.47 - Gram Stain of Haemophilus influenzae

Photo Credit: Deminorwood [CC BY-SA 4.0 (https://creativecommons.org/ licenses/by-sa/4.0)]

Figure 5.2.48 - Satellite colonies of Haemophilus influenzae growing near Staphylococcus aureus on blood agar

Figure 5.2.49 - Epiglottis

92

Photo Credit: [CC BY-SA 3.0 (https://creativecommons.org/licenses/bysa/3.0)]

Figure 5.2.51 - Cherry red epiglottis

Photo credit: Med Chaos [CC0]

Figure 5.2.50 - Thumb sign

93 REVIEW QUESTIONS 1. A researcher is studying an organism that demonstrates poor growth on 5% sheep blood agar. However, the same organism grows well when incubated next to colonies of Staphylococcus aureus. The organism described is known to cause invasive disease through the use of what virulence factor? A. B. C. D.

M protein Polyribosylribitol phosphate Erythrogenic exotoxin A Pyrrolidonyl arylamidase

• Correct answer: B • The organism being described is Haemophilus influenzae • Demonstrates poor growth on 5% sheep blood agar but grows well when grown next to colonies of Staphylococcus aureus • Staphylococcus aureus is β-hemolytic so it lyses red blood cells and provides nearby colonies of Haemophilus influenzae with NAD (factor V) and hematin (factor X) • The encapsulated strains of Haemophilus influenzae bacterium have a polyribosylribitol phosphate polysaccharide capsule • A is incorrect because M protein is a virulence factor associated with Streptococcus pyogenes (β-hemolytic and would grow well on blood agar) • C is incorrect because the erythrogenic exotoxin A is also associated with Streptococcus pyogenes and results in shock after it stimulates an overwhelming release of cytokines • D is false because pyrrolidonyl arylamidase (known as PYR) is associated with Streptococcus pyogenes and Enterococcus and is not a virulence factor

?

94 Section 25 - Bordetella pertussis

Border: Bordetella pertussis Pink colors: gram negative Boards blocking entrance: Bordet-Gengou agar Rays: Regan-Lowe medium G.I. Joe guy falling down with an arrow stuck in chest: the pertussis toxin disables the G subunit 6. Camping tent: ribosylation of the G subunit results in increased levels of cAMP 7. Another camping tent: adenylate cyclase toxin increases levels of cAMP 8. Tray and toxic gas: tracheal cytotoxin (damages respiratory epithelial cells) 9. Rope pulling catapult: filamentous hemagglutinin (FHA) facilitates the attachment of the pathogen to ciliated epithelial cells 10. Spherical rocks with a purple hue: causes a lymphocytosis 1. 2. 3. 4. 5.

i

i

11. Catapult: catarrhal stage 12. Guy coughing due to smoke: a mild cough is commonly seen during the catarrhal stage 13. Lamp: a fever is commonly seen during the catarrhal stage 14. Watery eyes next to onions: coryza is commonly seen during the catarrhal stage 15. Rocks: paroxysmal stage 16. Dust and severe coughing: severe coughing spells (paroxysmal cough) are commonly seen during the paroxysmal stage 17. Vomiting after coughing: post tussive vomiting may be seen during the paroxysmal stage 18. Convicts: convalescent stage 19. Zipper: treat with azithromycin 20. Syringe: vaccine (DTaP and Tdap) given to prevent infection

95

?

REVIEW QUESTIONS

1. A 2-year-old unvaccinated boy is brought to the emergency department due to difficulty breathing. His mother states that she first noticed his symptoms approximately two weeks ago which included a mild cough, a runny nose, and intermittent sneezing. Recently the cough has become so severe that he has had trouble breathing and has vomited several times afterward. Physical examination reveals episodes of severe coughing spells followed by a forced expiratory grunt. Laboratory analysis reveals a lymphocytosis. The most likely causal organism produces a toxin that:

Photo Credit: CDC Public Health Image Library [Public domain]

Figure 5.2.52 - Gram stain of Bordetella

A. B. C. D.

Inactivates the 60S ribosome Increases the activity of guanylate cyclase Increases the activity of adenylate cyclase Inactivates elongation factor 2

• Correct answer: C • This patient has a Bordetella pertussis infection • Difficulty breathing • Cough • Coryza • Posttussive emesis • Lymphocytosis • The pertussis toxin over activates adenylate cyclase by disabling G which results in increased levels of cyclic AMP • A is false because inactivation of the 60S ribosome is indicative of Shiga toxin and Shiga-like toxin from Shigella and E. coli • B is false because it is the heat-stable toxin of E. coli that increases the activity of guanylate cyclase • D is false because the toxins that inactivate elongation factor 2 are the diphtheria toxin of Corynebacterium diphtheriae and the exotoxin A of Pseudomonas aeruginosa i

Photo Credit: https://www.flickr.com/photos/54976525@N08/5905471297

Figure 5.2.53 - Regan-Lowe agar

96 Section 26 - Pasteurella multocida, Brucella spp., and Francisella tularensis

Bruce Wayne: Brucella Eiffel tower: Francisella tularensis Pastor: Pasteurella multocida Pink colors: gram negative Ejecting out of batmobile: Brucella species are facultative intracellular organisms 6. Rolling hills: Brucella is associated with an undulant fever 7. Fresh farmer’s market booth and milk: Brucella is associated with ingestion of unpasteurized dairy 8. Dirt bike: Brucella is treated with doxycycline 9. Rabbit: rabbits are reservoirs for Francisella 10. Tic tacs: ticks are a common reservoir for Francisella 11. Pavilion: Francisella is facultative intracellular 12. Joker cutting granny’s arm: Francisella causes painful skin lesions 13. Hand on head as if hot and sweating: Francisella causes a fever 14. Granny: Francisella is associated with granuloma formation 1. 2. 3. 4. 5.

15. Flies: deer flies are reservoirs for Francisella 16. Minnow fish: Francisella is treated with aminoglycosides 17. Pastor with blue necklace: Pasteurella is oxidase positive 18. Sack of bibles: Pasteurella has a polysaccharide capsule 19. Cat: Pasteurella is catalase positive 20. Dog biting pastor: Pasteurella is transmitted through dog bites 21. Blood from bite wound: Pasteurella grows on 5% blood agar 22. Wound on arm: Pasteurella is associated with animal bites and skin infections 23. Fish bones and scales: Pasteurella causes osteomyelitis 24. Penny: Pasteurella is treated with penicillin

97 REVIEW QUESTIONS

?

1. A 32-year-old female comes to the office due to a red swollen arm. She recently purchased an extremely hyper dog that bit her on the arm last night as they were playing. Immediately after the injury she felt fine so she went to sleep without much concern. However, when she woke up this morning she noticed the swelling and decided to come to the office. Physical examination is significant for an edematous and erythematous wound on the left arm that is tender to palpation. Cultures of the wound grow gram-negative coccobacilli that are catalasepositive. Which of the following is the definitive treatment for the most likely causal organism? Photo credit: CDC [Public domain]

Figure 5.2.54 - Gram stain of gram-negative coccobacilli

Photo Credit: Sanjay Mukhopadhyay [Public domain], via Wikimedia Commons

Figure 5.2.55 - Granuloma

A. B. C. D.

Daptomycin Doxycycline Gentamicin Penicillin

• Correct answer: D • The patient has a Pasteurella infection • Recent dog bite • Culture of the wound shows gramnegative coccobacilli that are catalasepositive • Standard treatment for Pasteurella is penicillin • A is false because daptomycin is commonly used to treat skin infections (MRSA), bacteremia, endocarditis, and VRE • B is false because Brucella is often treated with doxycycline but the commonly seen symptoms of undulant fever and weight loss, as well as a history of drinking unpasteurized milk are absent • C is false because gentamicin is used to treat Francisella tularensis

98 Section 27 - Salmonella spp.

Salmon: Salmonella Red and pink colors: gram negative Flag: flagellated Fish surrounded by a net: encapsulated Trail of blood: spreads hematogenously Pink bubbles that resemble neutrophils: polymorphonuclear neutrophils (PMNs) are seen in disseminated disease 7. Black cloud: hydrogen sulfide production on TSI agar 8. Meat and eggs: eggs and meat (poultry) are reservoirs 9. Turtle: turtles are reservoirs 10. Raspberry patch: invades the gastrointestinal tract through M cells (found in Peyer’s patches) 11. Pavilion: facultative intracellular 1. 2. 3. 4. 5. 6.

12. Lemons: acid-labile (large inoculum required) 13. Guy writing: may cause Reiter syndrome (reactive arthritis) 14. Vomiting: causes vomiting 15. Bloody water: causes bloody diarrhea 16. Three guys secreting the vomit: utilizes a type III secretion system 17. Toxic smoke cloud: one of the virulence factors of Salmonella is an endotoxin 18. Fish with prominent scales: causes osteomyelitis in patients with sickle cell disease 19. Trident: treat sickle cell osteomyelitis with ceftriaxone 20. “DO NOT PASS” sign: do not give antibiotics to patients with GI-related Salmonella infections

99 REVIEW QUESTIONS

Photo Credit: Roinujus [CC BY-SA 4.0 (https://creaivecommons.org/licenses/ by-sa/4.0)]

Figure 5.2.56 - Gram stain of Salmonella

Photo credit: Witmadrid [Public domain]

Figure 5.2.57 - TSI agar

Photo Credit: User:Plainpaper [CC BY-SA 3.0 (https://creativecommons.org/ licenses/by-sa/3.0)]

Figure 5.2.58 - Peyer’s patches

?

1. A 17-year-old female presents to the emergency department with a 4-hour history of nausea, vomiting, and diffuse abdominal pain. She also states that she has had several loose bowel movements and one appeared slightly red. Physical examination is significant for diffuse abdominal pain and a fever of 38.7°C. A stool sample is obtained and reveals oxidasenegative, nonlactose-fermenting organisms that produce a black color when placed in TSI agar. Treating the patient’s condition with ciprofloxacin would most likely result in: A. Rapid improvement of the patient’s condition B. Infertility C. An increased duration of bacterial fecal excretion D. Hepatosplenomegaly E. Osteomyelitis • Correct answer: C • The patient has gastroenteritis caused by Salmonella • Nausea, vomiting, and diffuse abdominal pain → gastroenteritis • Bloody diarrhea, oxidase-negative, nonlactose-fermenting, and black color on TSI agar → Salmonella • Treatment of Salmonella gastroenteritis with ciprofloxacin would increase the duration of the illness and an increased duration of bacterial fecal excretion would be seen (Salmonella gastroenteritis is usually self-limiting) • A is false because antibiotic treatment of Salmonella gastroenteritis is considered ineffective with respect to the duration of the illness • B and D are false because infertility and hepatosplenomegaly are not known side effects of ciprofloxacin (which can be toxic to the liver but does not affect the spleen) • E is false because osteomyelitis is a complication of Salmonella infection especially in patients with sickle cell disease who have areas of infarction and necrosis in certain bone and bone marrow locations

100 Section 28 - Salmonella typhi

Thai restaurant: Salmonella typhi Pink colors: gram negative Flag: flagellated Black smoke rising from eggs: hydrogen sulfide production on TSI agar 5. Green smoke: one of the virulence factors of Salmonella typhi is an endotoxin 6. Hiker guy: associated with a history of traveling 7. Sleep sack: has a Vi polysaccharide capsule 8. Lemons: acid-labile (large) inoculum required 9. Balloon that resembles a monocyte: immune response is primarily monocytes 10. Banana trees: can colonize the gallbladder in a chronic carrier state 1. 2. 3. 4.

11. Lamp: typhoid fever causes a fever 12. Blood stains: typhoid fever causes a rash characterized by rose spots on the abdomen 13. Guy sitting on toilet next to muddy brown river: typhoid fever causes abdominal pain, constipation, and diarrhea 14. Smelling flowers next to “LIVE FISH” sign: live attenuated vaccine 15. Syringe injecting juice into fish and juice spraying on sleep sack: the IM vaccine contains the Vi antigen from the polysaccharide capsule 16. Flower staff: treat with fluoroquinolones 17. Trident: treat with ceftriaxone

101 REVIEW QUESTIONS

Photo Credit: Content Provider(s): CDC/Armed Forces Institute of Pathology, Charles N. Farmer [Public domain]

Figure 5.2.59 - Rose spots

?

1. A 27-year-old male comes to the clinic due to fever, abdominal pain, and diarrhea. He first noticed the symptoms one week ago after returning from a humanitarian trip to Nepal. Physical examination reveals a temperature of 38.5°C, an erythematous maculopapular rash on his abdomen, and diffuse abdominal tenderness upon palpation. Blood and stool cultures are pending. This patient is most likely at risk of developing: A. Continued fecal bacterial shedding B. Rapid resolution of his symptoms without antibiotic administration C. Worsening of the rash which may involve the hands and palms D. Hemolytic uremic syndrome • Correct answer: A • The patient has an infection caused by Salmonella typhi • Fever, abdominal pain, diarrhea, recent travel history to an endemic area, and erythematous maculopapular rash on his abdomen • Patients who develop Salmonella typhi are at risk of developing a chronic carrier state of infection where the bacteria colonize in the gallbladder → continued fecal shedding of the bacteria as proliferation continues without treatment • B is false because the rapid resolution of symptoms without treatment is characteristic of nontyphoidal Salmonella species • C is false because of the rash of Salmonella typhi lasts for 10-14 days generally and is commonly distributed on the chest, back, and abdomen • D is false because Shigella and E. coli produce toxins that can cause HUS and damage the kidneys

102 Section 29 - Shigella

1. Shit: Shigella 2. Red sunset: gram negative 3. Pooper scooper team of three: utilizes a type III secretion system 4. Poop thrown from one person and landing on another person: human-to-human transmission only 5. Raspberry patches: invades the gastrointestinal tract through M cells (found in Peyer’s patches) 6. Horse reins that resemble actin: manipulates the host’s actin filaments which allows the organism to move intracellularly 7. Hoof prints that resemble the nuclei of neutrophils: immune response is primarily neutrophils 8. Stationary handcuffed guy: immotile (no flagella) 9. 60 minute news crew: Shiga toxin inhibits the 60S ribosomal subunit 10. Coal covered in blood: enterohemorrhagic E. coli (EHEC) has a toxin that closely resembles Shiga toxin (Shiga-like toxin) 11. Orange juice being blocked: acid resistant (low inoculum required)

12. Red candy on top of poop: Shiga toxin causes damage to the GI mucosa resulting in dysentery and bloody diarrhea 13. Tangled up strings that resemble the glomerulus and red balloons: Shiga toxin causes hemolytic uremic syndrome 14. Guy writing: may cause Reiter syndrome (reactive arthritis) 15. Shit and disco ball: Shigella dysenteriae (most severe) 16. Flexing guy: Shigella flexneri (2nd most severe) 17. Body builder guy: Shigella boydii (3rd most severe) 18. Body builder’s son: Shigella sonnei (4th most severe) 19. Amp: treated with ampicillin 20. Flower staff: treated with fluoroquinolones 21. Meth crystals: treated with TMP-SMX 22. Trident: treated with ceftriaxone

103

Photo Credit: Paulo Henrique Orlandi Mourao [CC BY-SA 3.0 (https:// creativecommons.org/licenses/by-sa/3.0)]

Figure 5.2.63 - Schistocytes

REVIEW QUESTIONS Photo Credit: CDC/Armed Forces Institute of Pathology (AFIP)

Figure 5.2.60 - Gram stain of Shigella

Photo Credit: Jeff16 [CC BY-SA 4.0 (https://creativecommons.org/licenses/ by-sa/4.0)]

Figure 5.2.61 - Actin

Photo Credit: Dr. Graham Beards [CC BY-SA 3.0 (https://creativecommons. org/licenses/by-sa/3.0)]

Figure 5.2.62 - Neutrophils

?

1. A 12-year-old girl is brought to the emergency department due to bloody diarrhea which began 2 days ago. Stool cultures grow nonlactose fermenting gram-negative bacilli that do not produce hydrogen sulfide when grown on triple sugar iron agar. It is determined that her infection is caused by an extremely invasive organism that can cause disease with exposure to as few as 10 organisms. The pathogenesis of this patient’s condition is most likely caused by: A. Hematogenous spread B. Invasion of cells that overlie intestinal lymphatic aggregates C. Aerosolized inhalation D. Excess proliferation of normal bacterial flora E. Spore germination • Correct answer: B • This patient has an infection caused by Shigella • Bloody diarrhea • Stool cultures that grow non-lactose fermenting gram-negative bacilli that do not produce HS on triple sugar iron agar • A very low inoculum is required for Shigella to cause disease • Shigella infects M cells which overlie the intestinal lymphatic aggregates (Peyer’s patches) • A is false because Shigella spreads from cell to cell - not hematogenously • C is false because Shigella infects after ingestion into the GI tract • D is false because Shigella doesn’t form part of the normal GI flora • E is false because Shigella isn’t sporeforming

104 Section 30 - E. coli

Coal mine: Escherichia coli (E. coli) Pink reflection: gram negative Bay: β-hemolytic Flowers: normal flora of the colon Green machine: green metallic sheen on eosin methylene blue (EMB) agar 6. Monkey animal crackers: E. coli is a lactose fermenter (pink color on MacConkey agar) 7. “Speed 50 MPH” sign: fast lactose fermenter 8. Cat: catalase positive 9. Dole whip: indole positive 10. Three guys helping carry water: utilizes a type III secretion system 11. Guy with bacteria spontaneously growing over his abdomen: causes spontaneous bacterial peritonitis 12. Pillar pipe on fire going to outhouse: E. coli has virulent pili that are associated with UTIs, cystitis, and pyelonephritis 13. Thin wires that resemble fimbriae: E. coli has virulent fimbriae that are associated with cystitis and pyelonephritis 14. Prostitute: causes prostatitis 15. Ammo near outhouse: amoxicillin can be used to treat E. coli UTIs 16. Sack and letter “K” on hat: K antigen is present in the polysaccharide capsule 1. 2. 3. 4. 5.

17. Pregnant woman and hat: serotypes of E. coli with the K antigen are commonly associated with neonatal meningitis 18. Pregnant lady coughing: serotypes of E. coli with the K antigen are commonly associated with pneumonia 19. Torch: a TORCHES infection 20. Lips, sack, sipping, and shock: has a lipopolysaccharide virulence factor that causes septic shock 21. Coal and blood: enterohemorrhagic E. coli (EHEC) 22. Pool of blood: EHEC causes bloody diarrhea 23. Microphone shaped like the number one: EHEC is the most common serotype of E. coli in the United States 24. Smashed cup of sorbet: EHEC does not ferment sorbitol 25. Hamburger and green leafy lettuce: EHEC is transmitted via undercooked meat and raw leafy vegetables 26. O157:H7 written on shirt: O157:H7 27. Shit hitting 60 minutes camera: EHEC produces shiga-like toxin that inhibits the 60S ribosomal subunit 28. Shit, strings that resembles the glomerulus, and red balloons: shiga-like toxin causes hemolytic-uremic syndrome

105 29. Broken plate: shiga-like toxin causes thrombocytopenia 30. Rock smashing balloon: hemolytic-uremic syndrome causes mechanical hemolysis resulting in anemia 31. Helmet shaped like a schistocyte: shiga-like toxin causes microthrombi formation --> schistocytes and renal failure 32. Rock on fire and rock with black spots: shiga-like toxin causes necrosis and inflammation 33. Rope pathway: enteropathogenic E. coli 34. Smashed plants that resemble intestinal villi: enteropathogenic E. coli adheres to apical surfaces of intestinal cells, flattens villi, and prevents absorption 35. Child’s clothes covered in mud: enteropathogenic E. coli causes diarrhea in children 36. Happy child with thumbs up on shirt: enteropathogenic E. coli does not produce a toxin 37. Vase and coal: enteroinvasive E. coli 38. Shit falling: the clinical manifestations of enteroinvasive E. coli are similar to Shigella

39. Rock on fire and rock with black spots: enteroinvasive E. coli invades the intestinal mucosa and causes inflammation and necrosis 40. Toxic cloud: enterotoxigenic E. coli 41. Travelers: enterotoxigenic E. coli causes travelers’ diarrhea 42. Water: enterotoxigenic E. coli causes watery diarrhea 43. Warm and stable guy: enterotoxigenic E. coli produces a heat-stable enterotoxin 44. Heat generator near warm and stable guy: the heatstable enterotoxin increases levels of cGMP 45. Emotionally-labile guy crushed by a rock: enterotoxigenic E. coli produces a heat-labile enterotoxin 46. Camping tent near emotionally labile guy: the heatlabile enterotoxin increases levels of cAMP 47. Fire getting put out by water: enterotoxigenic E. coli does not cause inflammation or invasion 48. Travelers with prominent collars: enterotoxigenic E. coli causes watery diarrhea that is similar to cholera

Photo Credit: Bobjgalindo [CC BY-SA 4.0(https://creativecommons.org/ licenses/by-sa/4.0)]

Figure 5.2.64 - Gram stain of E. coli

Photo Credit: Medimicro [Public domain]

Figure 5.2.66 - MacConkey agar

Photo Credit: Witmadrid [Public domain]

Figure 5.2.65 - Eosin methylene blue aga

106

Photo Credit: Microrao [Public domain]

Figure 5.2.67 - Indole test

Photo Credit: Botolph [Public domain]

Figure 5.2.68 - Lipopolysaccharide

107 REVIEW QUESTIONS 1. A 22-year-old male presents to the emergency department due to abdominal pain and diarrhea that started yesterday. He states that he recently ate an undercooked hamburger from a restaurant and is worried he may have developed an infection. Physical examination is significant for diffuse abdominal tenderness and guaiac-positive bloody stools. Stool cultures reveal a gram-negative organism that is indolepositive and does not ferment sorbitol on sorbitol-containing MacConkey agar. Additional laboratory analysis will most likely reveal which of the following? A. Thrombocytosis B. Elevated serum creatinine and blood urea nitrogen C. Fragmented platelets on a peripheral blood smear D. A positive Coombs test • Correct answer: B • The patient has an infection by enterohemorrhagic E. coli (EHEC) • Guaiac-positive bloody diarrhea • Does not ferment sorbitol • EHEC can cause HUS because the Shiga-like toxin causes microthrombi formation and results in schistocytes and kidney damage • Kidney damage results in elevated serum creatinine and blood urea nitrogen • A is false because E. coli does not cause thrombocytopenia via platelet consumption • C is false because the platelets are not fragmented, the RBCs are (schistocytes) • D is false because a positive Coombs test indicates the presence of antibodies that are causing RBC hemolysis which is not seen in E. coli infections

?

108 Section 31 - Klebsiella

1. Club: Klebsiella 2. Pink sunset: gram negative 3. Flowers next to dirty river: part of the normal gastrointestinal flora 4. Fast monkey: ferments lactose on MacConkey agar (fast lactose fermenter) 5. Sacks: polysaccharide capsule 6. Urine: urease positive 7. Spilled container of pills: associated with multidrug resistance 8. Mucous: mucoid colonies 9. Dyed beads and guy coughing: associated with lobar pneumonia in diabetics when aspirated

10. Molotov cocktail: associated with lobar pneumonia in alcoholics when aspirated 11. Strangled: may cause struvite kidney stones 12. Monkey eating liver: causes hepatic abscesses 13. Bacteria spontaneously growing on abdomen: causes spontaneous bacterial peritonitis 14. Young attractive girl next to urine: causes urinary tract infections 15. Red and green cubes of jello: classically associated with red “currant jelly” sputum 16. Black river: lung abscesses and liquefactive necrosis of lung tissue

109 REVIEW QUESTIONS

Photo Credit: Microrao, JJMMC, Davangere, Kamataka, India [Public domain]

Figure 5.2.69 - Mucoid colonies

?

1. A 53-year-old homeless male with a history of alcoholism is brought to the emergency department due to 3 days of chest pain, chills, and shortness of breath. He quickly destabilizes and passes away despite aggressive intervention. A sputum culture obtained upon admission to the emergency department revealed gram-negative bacilli that formed mucoid colonies. An autopsy revealed the presence of lung abscesses filled with liquefactive necrosis. The organism most likely responsible for this patient’s death: A. Slowly produces a pink color on MacConkey agar B. Rapidly produces a pink color on MacConkey agar C. Slowly produces a yellow color on MacConkey agar D. Rapidly produces a yellow color on MacConkey agar

Photo Credit: Doruk Salanci [CC BY-SA 3.0 (https://creativecommons.org/ licenses/by-sa/3.0)]

Figure 5.2.70 - Struvite crystals

• Correct answer: B • The patient has aspiration pneumonia due to Klebsiella • History of alcoholism, chest pain, and shortness of breath → aspiration pneumonia • Sputum culture shows gram-negative bacilli that form mucoid colonies and there are lung abscesses filled with liquefactive necrosis → Klebsiella infection • Klebsiella is a fast lactose fermenter on MacConkey agar (turns pink) • A is false because Klebsiella is a fast lactose fermenter • C and D are false because Klebsiella is a lactose fermenter and produces a pink color when plated on MacConkey agar

110 Section 32 - Proteus mirabilis

Proteus god: Proteus mirabilis Pink and red colors: gram negative Flipper: exhibits swarming motility when plated Urine: urease positive Black smoke near eggs: hydrogen sulfide production on TSI agar 6. Stones: kidney stones 1. 2. 3. 4. 5.

7. Strangled: may cause struvite kidney stones 8. Horns: staghorn calculi may be seen 9. Water splashing on girl’s crotch: causes urinary tract infections 10. Green stench cloud rising from fish: causes a distinct fishy scent 11. Ammo: treated with amoxicillin

111 REVIEW QUESTIONS

?

1. A 37-year-old male comes to the emergency department because of a 3-hour history of dysuria and hematuria. His temperature is 38.4°C, pulse is 112/min, and blood pressure is 158/84. Physical examination is significant for left costovertebral angle tenderness. Urinalysis reveals the presence of blood and a pH of 6.7 (normal: 39°C) 13. Ramming spike shaped like the Pontiac symbol: causes Pontiac fever 14. Salt spilling out of the damaged ship: associated with hyponatremia 15. Oil and gems spilling out of damaged ship: diagnosed by detecting antigens in the urine 16. Soldier using zip line: can be treated with azithromycin 17. Flowers wrapped around the mast: can be treated with fluoroquinolones

123 REVIEW QUESTIONS

Photo Credit: CDC-PHIL [Public domain]

Figure 5.2.82 - Gram stain of Legionella

?

1. A 51-year-old male with a history of chronic bronchitis is brought to the emergency department by his wife due to strange behavior for the past two days. She states that this morning he got lost driving home from the grocery store which is only a few blocks away from their home. She also states that he complained of diarrhea and a headache the day before. Physical examination is significant for a fever of 40.3°C (104°F). The patient is also mildly disoriented, but the remainder of the neurological exam is normal. A sputum gram stain reveals an abundance of neutrophils but no organisms. What additional finding will most likely be present in this individual? A. Gram-positive cocci colonization of the nares B. Antigens of the pathogen in the urine C. Hemolysis due to IgM autoantibodies D. Antibodies directed against hemagglutinin and neuraminidase

Photo Credit: William Cherry [Public domain]

Figure 5.2.83 - Silver stain of Legionella

Photo Credit: CDC/Megan Mathias and J. Todd Parker, Courtesy: Public Health Image Library

Figure 5.2.84 - Buffered charcoal yeast extract agar

• Correct answer: B • The patient has a Legionella infection • History of chronic bronchitis • CNS symptoms • GI symptoms • High fever • Abundance of neutrophils but no organisms seen on gram stain • Legionella can be diagnosed by detecting urine antigens • A is false because it is characteristic of Staphylococcus aureus • C is false because it describes the pathogenesis of Mycoplasma pneumonia • D is false because it refers to the mechanism of the Influenza virus

124 Section 38 - Campylobacter jejuni

1. Jiu jitsu at the battle of Iwo Jima: Campylobacter jejuni 2. Pink sky: gram negative 3. Curved mustache: curved bacilli 4. Blue necklace: oxidase positive 5. Man looking at poop on his shoe: fecal-oral transmission 6. American flag: flagellated (motile) 7. Hot explosion: grows at 42°C 8. Pig symbol on aircraft: pigs are reservoirs 9. Puppy: puppies are reservoirs

10. Stray cat: cats are reservoirs 11. Eating raw meat and drinking raw milk: transmitted through the ingestion of undercooked meat and unpasteurized milk 12. Injured men bleeding: causes bloody diarrhea 13. Green berets climbing hill, each one getting more exhausted toward the top: associated with GuillainBarré syndrome (ascending paralysis) 14. Reporter writing things down: associated with Reiter syndrome (reactive arthritis)

125 REVIEW QUESTIONS

?

1. A 6-year-old girl is brought to the physician due to bloody diarrhea that was first noticed yesterday. She has no significant past medical history and has not traveled recently. A stool culture reveals gram-negative curved bacilli that are oxidase-positive. The causal organism is most likely transmitted through which of the following?

Photo Credit: CDC [Public domain]

Figure 5.2.85 - Gram stain Campylobacter

A. B. C. D. E.

Contaminated water Sheep Sexual contact Pigs Respiratory droplets

• Correct answer: D • This patient has a Campylobacter jejuni infection • Bloody diarrhea • Stool culture reveals gram-negative curved bacilli that are oxidase-positive • Campylobacter jejuni can be transmitted through pigs • A is incorrect because although Vibrio cholera is a gram-negative curved bacilli that is oxidase positive and causes diarrhea, it doesn’t cause bloody diarrhea and is spread through contaminated water • B is a reference to Bacillus anthracis which is not associated with bloody diarrhea • C is incorrect because sexual contact is not typically associated with bloody diarrhea • E is a reference to many organisms such as Legionella

126 Section 39 - Vibrio cholerae & Vibrio vulnificus

Popped collar: Vibrio cholerae Pink sky: gram negative Curved mustache: curved bacillus Flag: flagellated Image of Africa: endemic to developing countries Acidic lemon juice: acid-labile (large inoculum required) 7. Air mattress pump: proton pump inhibitors increase the risk of infection 8. Blue necklace: oxidase positive 9. Contaminating water with old food: transmitted via ingestion of contaminated water 10. Flashlight powered by alkaline batteries: found in alkaline environments 11. Camping with tents: cholera toxin overactivates adenylate cyclase resulting in increased levels of cAMP 1. 2. 3. 4. 5. 6.

12. Knocking over pot of rice: causes “rice-water” diarrhea 13. Waterfall: causes watery diarrhea 14. Large jug of water: treat with oral rehydration solution 15. Volvo logo: Vibrio vulnificus 16. Monkey crackers: Vibrio vulnificus ferments lactose 17. Oysters: Vibrio vulnificus is associated with raw oyster consumption 18. Pack of alcohol: Vibrio vulnificus is associated with alcohol consumption 19. Sipping near Volvo: Vibrio vulnificus can cause sepsis 20. Dalmatian dog with a liver-shaped spot: Vibrio vulnificus can cause liver damage

127 REVIEW QUESTIONS

?

1. A resident physician is working in Ghana on a humanitarian trip when a 19-year-old male presents to the clinic with a 3-day history of watery diarrhea. He states that he recently drank some water that may have been contaminated. A stool sample reveals gramnegative, oxidase-positive, flagellated bacilli. The resident informs the medical team that his condition is caused by a toxin that: A. Invades the gastrointestinal mucosa B. Results in an absence of neutrophils in the stool C. Spreads hematogenously to vital organs D. Increases levels of cyclic GMP Photo Credit: CDC [Public domain]

Figure 5.2.86 - Gram stain of Vibrio cholerae

Photo Credit: F1jmm [CC BY-SA 3.0 (https://creativecommons.org/licenses/ by-sa/3.0)]

Figure 5.2.87 - “Rice-water” diarrhea

• Correct answer: B • This patient has a Vibrio cholerae infection • Recent travel exposure • Watery diarrhea • History of drinking contaminated water • Stool sample with gram-negative, oxidase-positive flagellated bacilli • Vibrio cholerae produces a toxin that results in rice-water diarrhea • The toxin is non-inflammatory and results in an absence of neutrophils in the stool as it does not invade the gastrointestinal mucosa • A is characteristic of the inflammatory diarrheal illnesses caused by Shigella or Salmonella • C is incorrect because this is describing Salmonella which can disseminate hematogenously • D is incorrect because Vibrio cholerae increases the levels of cyclic AMP - not cyclic GMP

128 Section 40 - Helicobacter pylori

1. 2. 3. 4. 5. 6. 7. 8. 9.

Pile of dirty clothes: Helicobacter pylori Pink walls: gram negative Curved mustache: curved bacillus American flag: flagellated (motile) Silver plate and silverware: can be diagnosed using silver-based staining methods Antlers: colonizes the antrum Cat on pile: catalase positive Cat urinating on dirty pile: urease positive Blue necklace: oxidase positive

Photo Credit: Ed Uthman, MD [CC BY-SA 2.0 (https://creativecommons.org/ licenses/by-sa/2.0)]

Figure 5.2.88 - Hematoxylin and eosin (H&E) stain of the gastric mucosa

10. Mattress dipping in the middle with a red spot: causes peptic ulcer disease 11. Green fart clouds: can cause chronic gastritis 12. Cancer hope ribbon: can cause gastric carcinoma 13. Malt shake: can cause MALT lymphoma 14. Green burp cloud: diagnosed with a urease breath test 15. Vacuum pump: treated with proton pump inhibitors 16. Ammo belt: treated with amoxicillin 17. Clarinet: treat with clarithromycin

Photo Credit: CDC/Dr. Edwin P. Ewing, Jr. [Public domain]

Figure 5.2.89 - Silver based stain

129

Figure 5.2.90 - Stomach anatomy and physiology

?

REVIEW QUESTIONS 1. A 27-year-old male with a history of chronic arthritis is brought to the emergency department after an episode of syncope which occurred an hour ago. He quickly regained consciousness, but has felt lightheaded since that time. Additional history reveals that he began having black tarry stools several days ago. His temperature is 37°C (98.6°F), pulse is 120/ min, and blood pressure is 80/60. This patient’s condition is most likely caused by which of the following? A. B. C. D.

Helicobacter pylori NSAID use Alcohol use Hypertriglyceridemia

• Correct answer: B • This patient has a peptic ulcer caused by chronic NSAID use • Chronic arthritis → likely taking NSAIDs • Recent syncope, black tarry stools, tachycardia, and hypotension → peptic ulcer and an upper GI bleed • Chronic NSAID use in a patient with a painful chronic condition is likely, and there is no information indicating an H. pylori infection in the question stem (answer A) • C is incorrect because while high concentrations of alcohol may damage the gastric mucosal barrier, there is no evidence that alcohol causes peptic ulcer disease • D is suggestive of pancreatitis

130 Section 41 - Mycobacterium tuberculosis

Tubers: Mycobacterium tuberculosis Microphone: mycolic acid Green acid: acid-fast bacillus Car bomb: carbol fuchsin LeBron James signature: can be grown in Löwenstein-Jensen medium 6. Smoke: enters the lower lobes of the lungs via aerosolized droplets 7. Cage: engulfed by alveolar macrophages and replicates intracellularly 8. Television cord: cord factor creates a “serpentine cord” appearance under the microscope 9. Tide bleach: produces a virulence factor known as sulfatide 10. Bleach on the floor in separate puddles: sulfatides prevent fusion of the phagosome and lysosome 11. Guy purifying clothes: purified protein derivative (PPD) test 12. Syringe: the BCG vaccine causes false positive PPD tests 13. Green rays: the IFN-γ release assay (IGRA) has fewer false positives among people with the BCG vaccine 14. Gong: a Ghon complex may form during primary TB 15. Person sleeping: latent TB 16. Granny: organisms are walled off via granuloma formation 1. 2. 3. 4. 5.

17. Fibrous ends of the blanket: the infected lung tissue becomes fibrotic 18. TNT remote: TNF-α inhibitors disrupt granuloma formation and may “awaken” the infection from its latent state 19. Emaciated kid: secondary TB occurs as health declines and the immune system becomes weaker 20. Upstairs: secondary TB affects the upper lobes of the lungs 21. Skinny: secondary TB presents with weight loss 22. Sweating: secondary TB presents with night sweats 23. Red beet juice near mouth: secondary TB presents with hemoptysis 24. Heater: secondary TB presents with a fever 25. Millet: miliary TB can occur following secondary TB or primary progressive TB 26. Pot of water: Pott disease may be seen in miliary TB 27. Hat with cavity due to weight of millet seeds: cavitary brain lesions may be seen in miliary TB 28. Smear of ripe food: treat primary, secondary, and miliary TB infections with RIPE therapy 29. Rifle and grandma holding a glass of ice: treat latent TB with rifampin and isoniazid

131

Photo Credit: CDC/Dr. George P. Kubica [Public domain], via Wikimedia Commons

Figure 5.2.91 - Acid-fast stain of Mycobacterium tuberculosis

Photo Credit: https://en.wikipedia.org/wiki/File:LJ_medium.JPG#file

Figure 5.2.92 - Lowenstein-Jensen medium

132

Figure 5.2.93 - Pathophysiology of Mycobacterium tuberculosis

133 REVIEW QUESTIONS

Photo Credit: Yale Rosen from USA [CC BY-SA 2.0 (https://creativecommons. org/licenses/by-sa2.0)]

Figure 5.2.94 - Miliary tuberculosis

?

1. A 57-year-old woman who emigrated to the U.S. from Thailand several years ago presents to the emergency department due to night sweats, fevers, and weight loss. She states that she first noticed her symptoms approximately one week ago. Her past medical history is significant for rheumatoid arthritis that is well controlled with infliximab. Physical examination shows axillary lymphadenopathy. A chest x-ray reveals a pulmonary infiltrate in the upper lobe of the left lung. This patient’s condition can be best explained by which of the following? A. Idiopathic pulmonary fibrosis B. Uncontrolled proliferation of ReedSternberg cells C. Granuloma dysfunction D. Polymorphonuclear invasion of alveolar tissue

Photo Credit: CDC/Ronald K. Smithwick [Public domain]

Figure 5.2.95 - “Serpentine cords”

Photo Credit: Greg Knobloch [Public domain]

Figure 5.2.96 - PPD test

Photo Credit: Basem Abbas Al Ubaidi [CC BY 4.0 (https://creativecommons. org/licenses/by/4.0)]

Figure 5.2.97 - Ghon complex

• Correct answer: C • This patient has reactivation of latent TB • From Thailand, night sweats, fever, weight loss, pulmonary infiltrate in the upper lobe of the left lung → suggestive of TB • Infliximab is a TNF-α inhibitor → granuloma breakdown → reactivation of latent TB • Granulomas form around latent TB through the action of macrophages which secrete TNF-α which helps form epithelioid macrophages and giant cells • Anti-TNF-α medications can cause sequestered granulomas to break down which results in the dissemination of the bacteria • A is incorrect because while idiopathic pulmonary fibrosis can cause shortness of breath, it’s unlikely to cause fever, night sweats, weight loss • B is describing cells that are associated with Hodgkin lymphoma • D is incorrect because this is describing pneumonia

134 Section 42 - Mycobacterium leprae

Leopards: Mycobacterium leprae Acid rain: acid-fast bacillus Car bombs: carbol-fuschin Microphone: mycolic acid Armadillo: armadillos are reservoirs in the U.S. Snow-capped mountains: prefers cool temperatures Prominent red gloves: causes superficial nerve infections resulting in a glove and stocking loss of sensation 8. Broken petri dish: cannot be grown in vitro 9. Biopsy dart: diagnosed with a skin biopsy 10. Three-chain fence: diagnosed using polymerase chain reaction 11. Two lazy leopards: the lepromatous form is associated with a weak immune response (T helper type 2 cells are activated) 12. Two leopards high on the hill near high amounts of green mold: the lepromatous form is associated with a high bacterial load 1. 2. 3. 4. 5. 6. 7.

13. Lion near the two lazy leopards: the lepromatous form is associated with leonine facies 14. Skull near the two lazy leopards: the lepromatous form can be lethal 15. Big tubercle: tuberculoid form 16. Caged leopard: the tuberculoid form is associated with a strong immune response (T helper type 1 cells are inactivated) 17. The cage and tubercle-ridden man are low on the hill near small amounts of green mold: the tuberculoid form is associated with a low bacterial load 18. Bald patches near tubercle: the tuberculoid form is associated with hairless skin plaques 19. Deputy: dapsone is used to treat both forms of leprosy 20. Rifle: rifampin is used to treat both forms of leprosy 21. Lazy leopard eating a clover: clofazimine is added for patients who have the lepromatous form

135

Photo Credit: U.S. Department of Health and Human Services [Public domain]

Figure 5.2.98 - Acid-fast stain of Mycobacterium leprae

Figure 5.2.99 - Pathophysiology of Mycobacterium leprae

136

Photo Credit: Rosemary A. Jones [CC BY-SA 4.0 (https://creativecommons.org/licenses/by-sa/4.0)]

Figure 5.2.100 - Leonine Facies

137

?

REVIEW QUESTIONS 1. A 41-year-old male who recently emigrated from Nigeria comes to the office due to a skin lesion on his head. He states that he first noticed the area a few weeks ago. Physical examination reveals a hypoesthetic, hypopigmented lesion on the right side of the patient’s forehead. Adjacent to the lesion is an area of baldness extending several centimeters into his hairline. A biopsy is obtained and light microscopy reveals numerous pathogens invading the nerve tissue. Which of the following is true regarding this patient’s condition? A. There is an abundance of Th2 cells in response to the infection B. The cell wall of the pathogen contains many long fatty acids C. The causal organism prefers warm temperatures D. IL-4 is the cytokine involved in this immune response

• Correct answer: B • The patient has the tuberculoid form of leprosy • From Nigeria • Hypoesthetic, hypopigmented lesion as well as with an area of baldness → tuberculoid form of leprosy • Mycobacterium leprae invades the skin and nerves • The cell wall of the bacterium has many long fatty acids which is describing mycolic acid found in Mycobacterium leprae • A is incorrect because this patient has the tuberculoid form of leprosy which is characterized by a Th1 response - not a Th2 response • C is not characteristic of Mycobacterium leprae (it prefers cool temperatures) • D is incorrect because IL-4 is more involved with Th2 responses

138 Section 43 - Mycobacterium avium-intracellulare, Scrofulaceum, and Marinum

1. Aviary: Mycobacterium avium-intracellulare (MAC) 2. Child with a scruffy beard: Mycobacterium scrofulaceum 3. Marine biologist bitten by piranha: Mycobacterium marinum 4. Acid pond: acid-fast 5. Microphone: mycolic acid 6. Man in the aviary has a Band-Aid: MAC causes disease in AIDS patients 7. Bead-like segments in the aviary: MAC causes lymphadenitis 8. Heat lamp in the aviary: MAC causes a fever 9. Sweating man within the aviary: MAC causes night sweats 10. Skinny man within the aviary: MAC causes weight loss

11. Man using a zip line and wearing a shirt with all 50 states: azithromycin is given when the CD4 count is < 50 12. Spilled medicine container: MAC is often resistant to multiple drugs 13. Scruffy beard on a child: M. scrofulaceum is common in children 14. Scruffy child holding a beaded net near his neck: M. scrofulaceum causes cervical lymphadenitis in children 15. Marine biologist working closely with piranhas: M. marinum causes hand infections in aquarium handlers

139 REVIEW QUESTIONS

?

1. A 33-year-old male with a history of HIV presents to the physician due to weight loss, fever, and a swollen lump in his neck. His temperature is 38.8°C (101.8°F). Physical examination reveals a swollen and tender cervical lymph node. A tissue aspirate of the swollen lymph node reveals acid-fast bacilli. A CD4 count is obtained and will most likely reveal which of the following? Photo Credit: CDC/Dr. Edwin P. Ewing, Jr. [Public domain]

Figure 5.2.101 - Acid-fast stain of Mycobacterium avium-intracellulare

A. B. C. D. E.

43 cells/microL 97 cells/microL 439 cells/microL 991 cells/microL 1,503 cells/microL

• Correct answer: A • The patient has MAC • Weight loss • Fever • Cervical lymphadenitis • Tissue aspirate revealed acid-fast bacilli • HIV diagnosis makes an AIDS-defining illness such as MAC more likely than the rare M. scrofulaceaum • MAC infections are most common when the CD4 count drops below 50 which is why these patients may be given prophylactic azithromycin • B-E are all >50 cells/microL

140 Section 44 - Borrelia burgdorferi

1. Burglar: Borrelia burgdorferi 2. Spiral staircase: spirochete 3. “Northeast corner” sign in the northeast part of the image: common in the northeastern part of the U.S. 4. Tic tacs near exit sign: transmitted through the Ixodes tick 5. Anne of Green Gables: the Ixodes deer tick is also a vector for Anaplasma and Babesia 6. Games: can be visualized using the Giemsa stain 7. Deer chasing a mouse: deer and mice are reservoirs and important to the tick life cycle 8. Target: stage 1 is associated with a “bulls-eye” rash known as erythema migrans 9. Sweat: stage 1 is associated with flu-like symptoms

10. Bell: stage 2 is associated with Bell’s palsy 11. Car on fire: stage 2 is associated with carditis 12. Blocking customers from car store: stage 2 is associated with atrioventricular block 13. Red knees and elbows: stage 3 is associated with migratory arthritis 14. Water all over guy’s head: stage 3 is associated with encephalitis 15. Dirt bike: treat with doxycycline 16. Ammo near pregnant woman and child: treat with amoxicillin (pregnant women and children) 17. Cephalopod toy: treat with 2nd generation cephalosporins (pregnant women and children)

Photo Credit: Content Provider(s): CDC [Public domain]

Photo Credit: Courtesy of the U.S. Department of Health and Human Services

Figure 5.2.102 - Spirochetes via dark-field microscopy

Figure 5.2.103 -  Ixodes deer tick

141

Photo Credit: Alan R. Walker [CC BY-SA 3.0 (https://creativecommons.org/ licenses/by-sa/3.0)]

Figure 5.2.104 - Giemsa stain of Borrelia

Photo Credit: https://upload.wikimedia.org/wikipedia/commons/1/1d/ Bellspalsy.JPG

Figure 5.2.106 - Bell’s palsy

REVIEW QUESTIONS

?

1. A 50-year-old female presents to the physician due to a rash on her leg that she first noticed yesterday. She also states that she has felt general malaise since returning home from a hiking trip to the Appalachian mountains three days ago. Physical examination reveals a rash on the left leg with a central area of erythema and a surrounding erythematous ring. If left untreated, which of the following complications is most likely to occur in this patient? A. B. C. D. E. Photo Credit: James Gathany, Content Provider(s): CDC/James Gathany [Public domain]

Figure 5.2.105 - Erythema migrans

Pneumonia Meningitis Arthritis Hepatitis Aplastic anemia

• Correct answer: C • This patient has erythema migrans associated with stage 1 of Lyme disease • Recently travel to the Appalachian mountains (east coast of the U.S.) • Rash on the left leg with a central area of erythema and a surrounding erythematous ring → erythema migrans (bulls-eye rash) → stage 1 • Migratory arthritis is a common complication if left untreated • A, B, D, and E are not commonly associated with Lyme disease

142 Section 45 - Leptospira interrogans

1. Spear: Leptospira interrogans 2. Spiral waves: spirochete 3. Fishing hook near contaminated water: hook-shaped ends commonly found in water contaminated with animal urine 4. Tropical trees: leptospirosis is prevalent in the tropics 5. Surfer: leptospirosis is prevalent among surfers 6. Yellow surfboard: leptospirosis can cause jaundice 7. Sunglasses and looking away from the sun: leptospirosis can cause photophobia 8. Blood spattered on sunglasses: leptospirosis can cause conjunctival erythema

9. Sweating surfer: leptospirosis causes flu-like symptoms including myalgias 10. Whale: Weil disease 11. Dalmatian with liver-shaped spot: Weil disease can cause liver dysfunction 12. Yellow surfboard: Weil disease can cause jaundice 13. Heat lamp: Weil disease can cause a fever 14. Uncontrolled bleeding: Weil disease can cause hemorrhage and anemia 15. Kidney-shaped shrimp: Weil disease can cause renal failure

143 REVIEW QUESTIONS 1. A 24-year-old male presents to the physician due to myalgias and a headache. He states that he first noticed his symptoms after returning home from a surfing trip to Hawaii two days ago. Physical examination reveals conjunctival erythema. The physician suspects a spirochete infection and immediately begins antibiotic therapy. Which of the following complications is most likely associated with this patient’s condition? Photo Credit: CDC Public Health Image Library. CDC/NCID/HIP/Janice Carr (PHIL #1220)

Figure 5.2.107 - Electron microscopic image of Leptospira interrogans

A. B. C. D.

A painful genital ulcer A maculopapular rash on the trunk A painless genital ulcer Renal failure

• Correct answer: D • This patient has an infection caused by Leptospira interrogans • Surfing in Hawaii • Developed myalgias, a headache, and conjunctival erythema • A spirochete infection is suspected → Leptospira interrogans is the likely cause • Weil disease can cause renal failure in individuals infected with Leptospira interrogans • A is associated with Haemophilus ducreyi • B is incorrect as it is associated syphilis which has a rash that affects the palms and soles • C is describing the classic lesion seen in syphilis (painless chancre)

?

144 Section 46 - Treponema pallidum

1. Paladin: Treponema pallidum 2. Spiral staircase: spirochete 3. Fighting dark creature with magic: dark field microscopy can identify Treponema pallidum 4. Armor with the letter “S”: sexually transmitted infection 5. Hammer: Jarisch-Herxheimer reaction 6. Skull symbol on the hammer: Jarisch-Herxheimer reaction occurs following the destruction of the organism 7. Hammer embedded with copper pennies: symptoms associated with the Jarisch-Herxheimer reaction begin following treatment with penicillin 8. Shielding cane attack: primary syphilis presents with a painless genital chancre 9. Spilling vitriol: VDRL test 10. Tombstone reading “RIP”: RPR test 11. Heart and bones that resemble phospholipids are poking up from the ground near the vitriol and “RIP” tombstone: the presence of anticardiolipin antibodies indicates a positive test result 12. Fire arrow shot between the goblin’s eyes: the FTAABS test is the “confirmatory” test 13. Palms and soles with hair and bumps: secondary syphilis may present with a maculopapular rash involving the trunk, palms, and soles

14. Lumpy flail: secondary syphilis may present with a wart-like lesion on the genitals known as condyloma lata 15. Beaded net: secondary syphilis may present with lymphadenopathy 16. Bald patches on mangy monster: secondary syphilis may present with patchy hair loss 17. Grandpa-looking wizard: tertiary syphilis may present with gummas 18. Gannondolf wearing a hat: neurosyphilis is a common complication of tertiary syphilis 19. Damaged columns: tertiary syphilis causes tabes dorsalis (demyelination of the nerves within the dorsal column) 20. Aorta-shaped tree: tertiary syphilis may cause aortitis resulting in a tree barking appearance of the aorta 21. Eyes appearing angry and focused: tertiary syphilis may present with Argyll Robertson pupils 22. Hobbit with swollen red knees: tertiary syphilis may present with neuropathic arthropathy (Charcot joint) 23. Baby Balrog: congenital syphilis 24. Balrog’s notched teeth: congenital syphilis may cause notched teeth 25. Intricate horns near the ears: congenital syphilis may cause hearing loss

145 26. Odd-shaped nose: congenital syphilis may cause saddle nose 27. Fire coming out of mouth, nose, and head: congenital syphilis may cause snuffles (nasal discharge laden with Treponema pallidum) 28. Wide openings near the sides of the mouth: congenital syphilis may cause rhagades (wrinkled skin and linear scars at the angle of the mouth)

29. Deformed shins: congenital syphilis may cause saber shins 30. Fighting the Balrogs using a penny-based spell: congenital syphilis can be prevented if the mother is treated with penicillin early during pregnancy 31. Pennies on the hammer and used against the Balrog: syphilis is treated with penicillin

Photo Credit: Herbert L. Fred, MD, Hendrik A. van Dijk [CC BY-SA 3.0 (https:// creativecommons.org/licenses/by-sa/3.0)] Photo Credit: Content Providers: CDC/C.W. Hubbard [Public domain]

Figure 5.2.108 - Dark-field photomicrograph of Treponema pallidum

Figure 5.2.110 - Maculopapular rash

Photo Credit: CDC/Robert Sumpter [Public domain] Photo Credit: Courtesy of the U.S. Department of Health and Human Services and M.Rein

Figure 5.2.109 - Chancre

Figure 5.2.111 - Maculopapular rash on the palms

146

Photo Credit: Courtesy of the U.S. Department of Health and Human Services and Susan Lindsley

Figure 5.2.112 - Condyloma lata

Photo Credit: Courtesy of the U.S. Department of Health and Humna Services and Susan Lindsley

Figure 5.2.115 - Notched teeth

Photo Credit: CDC [Public domain]

Figure 5.2.113 - Gumma

Figure 5.2.114 - Cross section of the spinal cord

147 REVIEW QUESTIONS

?

1. A 23-year-old male comes to the physician due to a small painless lesion on his penis that he noticed yesterday. He is concerned he may have a sexually transmitted infection because 4 days ago he had unprotected sexual intercourse with a prostitute. Physical examination reveals a painless ulcer on the shaft of the penis. A serum Venereal Disease Research Laboratory (VDRL) test is negative. Which of the following should be performed to make an accurate diagnosis of this patient’s condition? A. The rapid plasma reagin (RPR) test B. Repeat the Venereal Disease Research Laboratory (VDRL) test C. The Toluidine Red Unheated Serum Test (TRUST) D. The fluorescent treponemal antibody absorption (FTA-ABS) test

Photo Credit: https://en.m.wikipedia.org/wiki/File:Saddle_nose_38.jpg

Figure 5.2.116 - Saddle nose

Photo Credit: Courtesy of the U.S. Department of Health and Humnan Services and Robert Sumpter

Figure 5.2.117 - Rhagades

• Correct answer: D • This patient has a painless lesion on his penis (chancre), which is a feature of primary syphilis • The VDRL is negative but this does not rule out Treponema pallidum infection due to the time it takes to develop an immune response (about 4 weeks) • False negatives are common early in the course of infection • The fluorescent treponemal antibody absorption (FTA-ABS) test is a confirmatory test that detects the presence of Treponema pallidum antigens and should be performed in this case • Always remember to perform both a nontreponemal test (VDRL or RPR) and a treponemal test (FTA-ABS) when diagnosing cases of syphilis • Answers A-C are incorrect because they are all nontreponemal tests which are based upon the detection of anticardiolipin antibodies (the patient already had the nontreponemal VDRL test so it would be pointless to perform another test with a similar mechanism)

148 Section 47 - Chlamydia trachomatis, Chlamydophila pneumoniae, and Chlamydophila psittaci

1. Clams: Chlamydia 2. Letter “S” on shirt: most serotypes of  Chlamydia trachomatis are sexually transmitted 3. Cage: obligate intracellular (cannot make own ATP) 4. Hugging elementary school kids: elementary bodies enter the cell via endocytosis 5. Rakes inside school: Reticulate bodies replicate inside of the cell by fission 6. Kids leaving elementary school: Reticulate bodies are reorganized into elementary bodies, released from the cell, and then infect adjacent cells 7. Broken mirror: the cell wall lacks muramic acid 8. Games: identified using the Giemsa stain 9. Gnats: diagnosed with NAAT (amplifies bacterial DNA or RNA sequences using PCR) 10. Apples: serotypes A, B, and C 11. Dog: serotypes D-K 12. Lamp: serotypes L1, L2, and L3 13. Holding hands: serotypes A, B, and C exhibit handto-hand transmission 14. Blind African American man: serotypes A, B, and C are common in Africa and can cause trachoma 15. Pelvis on fire: serotypes D-K cause urethritis and PID 16. Pregnant woman partially inside and outside of car: serotypes D-K cause PID which increases the risk of ectopic pregnancy

17. Red car fragments and coughing babies: serotypes D-K cause neonatal pneumonia with eosinophilia 18. Two babies with red eyes: serotypes D-K cause neonatal conjunctivitis 1-2 weeks after birth 19. Limping granny: serotypes L1, L2, and L3 cause lymphogranuloma venereum 20. Cup: serotypes L1, L2, and L3 cause painless genital ulcers 21. Swollen lesions on side of legs: serotypes L1, L2, and L3 cause painful inguinal buboes 22. Dirt bike: serotypes L1, L2, and L3 are treated with doxycycline 23. Writing: Reiter syndrome 24. Gondola toys: Gonorrhoeae coinfection 25. New mower with clam logo: Chlamydophila pneumoniae 26. Pistachios: Chlamydophila psittaci 27. Smoke: C. pneumoniae and C. psittaci are transmitted through aerosolized droplets 28. Coughing guys: C. pneumoniae and C. psittaci cause pneumonia 29. Blue bird: birds are reservoirs for C. psittaci 30. Trident: treat with ceftriaxone 31. Zip-line: treat with azithromycin

149 REVIEW QUESTIONS

?

1. A 12-day-old infant is brought to the emergency department by his mother due to red eyes. She states that she did not have prenatal care due to financial concerns, but that the boy was delivered at home without complications. Physical examination reveals erythema and mucopurulent discharge surrounding both eyes. The infectious serotype responsible for this patient’s condition may also cause: A. B. C. D. Photo Credit: Huckfinne [Public domain]

Figure 5.2.118 - Life cycle of Chlamydia trachomatis

Photo Credit: Doctors E. Arum and N. Jacobs [Public domain]

Figure 5.2.119 - Giemsa stain of Chlamydia

Groin pain and a painless genital ulcer Trachoma Septic arthritis Pelvic inflammatory disease

• Correct answer: D • This patient has neonatal conjunctivitis caused by Chlamydia trachomatis • Normal onset is 5-14 days after birth • Specifically caused by serotypes D-K • Serotypes D-K also cause pelvic inflammatory disease • A is incorrect because it describes lymphogranuloma venereum which is caused by serotypes L1, L2, and L3 • B is incorrect because this is the leading cause of blindness in the world and is caused by serotypes A, B, and C • C is incorrect because it is a complication associated with Gonorrhea but Gonorrhea causing neonatal conjunctivitis typically occurs 2-5 days after birth

150 Section 48 - Coxiella burnetii

1. 2. 3. 4. 5. 6. 7. 8.

Brunette queen: Coxiella burnetii Pink sunset: gram negative Queens crown: causes Q fever Cow placenta: transmitted through the aerosols of cattle amniotic fluid Sheep wool: transmitted through the aerosols of sheep amniotic fluid Plate of snails: a sporulation-like process protects the organism from harsh environmental conditions Caged sheep: obligate intracellular organism Black horses: a rash is rarely seen in patients with Q fever

9. Coughing stepsister: patients with Q fever may present with pneumonia 10. Liver-shaped spot: patients with Q fever may present with hepatitis and transaminitis 11. Sweating stepsister: patients with Q fever may present with flu-like symptoms 12. Broken plate: patients with Q fever may present with thrombocytopenia 13. Broken petri dish near cart: patients with Q fever may present with culture-negative endocarditis 14. Dirt bike: treat with doxycycline

151 REVIEW QUESTIONS

Photo Credit: CDC Public Health Image Library [Public domain]

Figure 5.2.120 -  Gram stain of Coxiella burnetii

?

1. A 27-year-old male presents to the emergency department due to a subjective fever, myalgias, and shortness of breath for the past several days. He has also noticed strange bumps on the pads of his fingers that are quite tender. When asked about recent changes in his life he states that approximately one month ago he was hired to work on a dairy farm. Physical examination reveals a cardiac murmur and tender raised lesions on his finger pads. Blood cultures are obtained but no organisms are grown. Which of the following additional findings would most likely be observed in this patient? A. An AST of 19 U/L (normal: 8-20 U/L) B. A platelet count of 73,000/mm3 (normal: 150,000-400,000/mm3) C. A hemoglobin of 10.4 g/dL (normal: 13.517.5 g/dL) D. A chest radiograph with bilateral hilar adenopathy

Photo Credit: Roberto J. Galindo [CC BY-SA 4.0 (https://creativecommons.org/ licenses/by-sa/4.0)]

Figure 5.2.121 -  Osler’s nodes

• Correct answer: B • This patient has Q fever • Subjective fever • Myalgias • Shortness of breath • History of working on a dairy farm • Osler nodes and negative blood cultures → culture-negative bacterial endocarditis • Patients with Q fever may have thrombocytopenia • A is incorrect because Q fever also causes hepatitis so a normal AST would be less likely than an elevated AST • C is incorrect because anemia is an extremely rare finding in patients with Q fever • D is incorrect because while fever can cause pneumonia, this description is suggestive of sarcoidosis - not pneumonia

152 Section 49 - Gardnerella vaginalis

1. 2. 3. 4. 5. 6.

Garden: Gardnerella vaginalis Clue: clue cells Decayed skin edges: stippled edges Gray skin: grayish vaginal discharge Fish: fishy smell Vagina shaped flower on shield: painless vaginitis

7. Roses: inflammation caused by overgrowth of normal vaginal flora 8. Blue and red garden bugs: gram variable 9. Cleaning lady: clindamycin as treatment 10. Bleach: vaginal pH>4.5 (alkaline) 11. Metro: metronidazole as treatment

153 REVIEW QUESTIONS

?

1. A 29-year-old female presents to the physician complaining of vaginal discharge with a fishy odor for the past 4 days. The physician obtains a sample of the discharge for laboratory tests. Following microscopic examination, the lab does not identify any fungi, protozoa, or bacteria foreign to the vagina. Epithelial cells covered with normal vaginal flora were seen. What is the likely pH range of the patient’s vaginal discharge?

Photo Credit: CDC/M. Rein [Public domain], via Wikimedia Commons

Figure 5.2.122 - Clue cells

• Correct answer: pH > 4.5 • Gardnerella vaginalis infection is suspected due to the following: • Vaginal discharge with a fishy odor • Clue cells (epithelial cells covered with normal vaginal flora) • Lack of fungi makes Candidal vaginitis unlikely • Lack of protozoa makes Trichomonas vaginalis unlikely

154 Section 50 - Mycoplasma pneumoniae

Plasma alien: Mycoplasma pneumoniae Blobby mass: Mycoplasma lacks a cell wall Coughing: causes pneumonia Eaten man: grows on Eaton agar College dorm: outbreaks common with young adults (